Vous êtes sur la page 1sur 136

Reading Step 2

 Vocabulary Test

Vocabulary back to top

The first kind is vocabulary. It’s very easy to identity. Whenever the question asks “what
does this this word mean,” it’s a vocabulary question. Here is an example to familiarize
yourself with this question type:

The word “depressions” in the passage is closest in meaning to

If you know the word:

1)    Choose the answer or answers that match the meaning best.

2)    Read the sentence with the highlighted vocabulary to double check the
meaning.

If you don’t know the word:

1)    Read the sentence with the word and guess what its meaning could be

2)    Choose an answer that fits this meaning. (If more than one works, guess fast.)
 

You shouldn’t spend a lot of time on this question no matter what. If you don’t know the
word, more than one answer choice might work and you’ll just waste time thinking. Just
guess and move on to spend your time on a different question.

Reading Practice back to top

Cave Paintings in Lascaux

The Lascaux Grotto, in Southwest France, is a series of narrow cave chambers that was
discovered by children playing in the 1940s. ❒ These caves contained an amazing find
for historians – prehistoric paintings of animals on the walls. ❒ These massive animals
are sometimes portrayed as following one after another in a long line, but on other
occasions, they are sideways on the wall and some even upside down. Paintings are
mostly done in reds, yellow, and brown, and feature such beasts as wild horses, bulls,
reindeer, bison, and mammoths. ❒ Scientists analyzed the substance of the “paint” and
discovered that the colors came from ocher and other iron oxides that had been ground
into a fine powder. ❒ This powder then appears to have been brushed or smeared onto
the cave walls in some cases, while in other cases the artists of these paintings seem to
have blown or sprayed the material onto the rock surface. In these cases, scientists
surmise that tubes made from hollow bone may have been used as tools for the spraying
because hollow bones marked with pigment were discovered close by.

The location of these particular paintings has become a source of mystery to the
scientific community. ❒ Unlike rock paintings found in other parts of the world, these
were not located near the entrance to the cave or completely out in the open. ❒ Instead,
they are deep within the caverns, quite a distance from the opening to the grotto and any
natural light. ❒ In addition, it seems likely that the drawers of these paintings did not
necessarily want them to be seen by others, as inhabitants of caves usually lived
relatively close to the entrances. ❒ The odd location of these particular paintings has led
to various theories as to why so many generations of artists chose to keep their work so
well hidden.

❒ Three opinions have been given by the scientific community as to the perplexing
origins and meaning of these paintings. ❒ The most straightforward of these
explanations is that the paintings are simply a record of the migrations of herds of local
animals. ❒ And since some paintings are drawn directly over others, scientists theorize
that the utility of each painting was over when the migration it depicted had ended. ❒
However, this theory fails to address the puzzling location of these paintings within the
caves, unless it was simply that the cave dwellers chose to commemorate the migrations
with ceremonies held in secret.

A second theory holds that these paintings were tied to hunting and they played a vital
role in a special preparation ceremony for the hunters. ❒ The theory states that the
paintings were part of a ritual used to prepare the hunters psychologically for the
challenges they were about to face. ❒ The fact that in several of the paintings the
animals are portrayed as wounded by arrows or spears gives particular credence to this
theory. ❒ And the overpainting is explained by the fact that the hunters had no use for
the paintings after the hunt itself, and thus felt free to cover them up in preparation for
their next foray into the field. ❒

A third opinion on the paintings offers the possibility that the animals portrayed on the
walls may have assumed some sort of mythical meaning to the cave dwellers, either as
ancient ancestors or as protectors of the members of the tribe. Supporters of this theory
point to two different pieces of evidence. First, there are the strange and cryptic
geometric shapes painted near the animals. Second, there is the remarkable absence of
men in any of the paintings. In the few cases that humans are depicted, they are very
crudely drawn with long and rigid bodies. ❒Some of the men are shown lying face down
while others have the heads of birds and other animals. ❒This suggests to advocates of
the theory that the people in the picture might be portrayed in a trance state.
❒However, there is no definitive proof for this theory and it could just as easily be that
these bird-headed men are simply wearing masks for hunting.❒

While these theories continue to compete for primacy in the scientific community, and
there is still heavy debate. It is possible that there will never be a satisfactory answer to
this intriguing question. However, it is clear that the Lascaux paintings represent a great
artistic achievement for the time that will continue to engage researchers, whatever their
significance may be.

1)    The word “smeared” in paragraph 1 is closest in meaning to

a)    dabbed

b)    splattered

c)    thrust

d)    spread

2)    The word “surmise” in paragraph 1 is closest in meaning to

a)    propose
b)    recommend

c)    insinuate

d)    infer

3)    The word “grotto” in the paragraph 2 is closest in meaning to

a)    cave

b)    space

c)    home

d)    chamber

4)    The word “inhabitants” in the paragraph 2 is closest in meaning to

a)    interlopers

b)    intruders

c)    guests

d)    residents

5)    The word “perplexing” in paragraph 3 is closest in meaning to

a)    outstanding

b)    inconceivable
c)    puzzling

d)    endearing

6)    The word “migrations” in paragraph 3 is closest in meaning to

a)    homecomings

b)    movements

c)    transferences

d)    mitigations

7)    The word “credence” in paragraph 4 is closest in meaning to

a)    believability

b)    importance

c)    substance

d)    guile

8)    The word “foray” in paragraph 4 is closest in meaning to

a)    parade

b)    tour

c)    venture
d)    march

9)    The word “rigid” in paragraph 5 is closest in meaning to

a)    limp

b)    turgid

c)    stiff

d)    flowing

10)    The word “portrayed” in paragraph 5 is closest in meaning to

a)    depicted

b)    crowded

c)    included

d)    established

Power Review back to top

Now, let’s review by reinforcing strategy and the facts about getting a higher score on
the TOEFL.

 
Question 1

This is a vocabulary question. These questions are both easy and difficult. Very
easy because if you know the answer, you find the answer easily. They are difficult
because when you don't know what the word means, it can be impossible to learn
what it means from reading.

If you don’t know the word:

1)    Read the sentence with the word.

2)    Look for clues to help find the meaning and guess what its meaning
could be.

3)    Choose an answer that fits this meaning. (If more than one works, guess
fast.)

1)    First, we see the word smeared in the sentence that it appears in: This
powder then appears to have been brushed or smeared onto the
cave walls in some cases, while in other cases the artist of these
paintings seem to have blown or sprayed the material onto the
rock surface.

2)    We see next to our vocabulary word, the word or. Or shows similarity.
We can assume that smeared and brushed, therefore, have a similar
meeting. Of course, it doesn’t have to be, but we’re guessing. We also have an
extra clue; we have the word while. While shows contrast, so if we read
that sentence, we know that smeared should have the opposite meaning of
the information we see in the while clause: while in other cases the
artist of these painting seem to have blown or sprayed the
material. Smeared should be the opposite of blown or sprayed.

3)    We then use that information to guess at the answer. Dabbed means to
tap lightly. Splatter means to throw on randomly. Thrust means to push or
force on. Spread means to place across. Smear means to place across. We
know our answer is then, D.

Question 2

This is a vocabulary question and we know this because the question asks us to
identify an answer that is closest in meaning to a word. Follow strategy when you
don’t know it and choose the right answer.

1)    In the sentence with surmise, we read: scientists surmise that
tubes made from hollow bone may have been used as tools . . .

2)    This is a tough vocabulary question because of the answer choices. What
do scientists do to form an idea? We guess our answer: think, guess, form,
create, hypothesis, etc.

 
3)    Unfortunately, in this case, each answer choice has a similar meaning to
what we expect and so we must guess quickly and move on if we don’t know
the meaning. The actual answer is D. Memorize this if you don't know the
word.

*Remember that if there are several vocabulary words in this reading that you
don’t know. You must start to build your vocabulary daily any way that you can
and we encourage you to participate in our vocabulary program, which is designed
to help with the TOEFL reading.

Question 3  

This is a vocabulary question and we know this because the question asks us to
identify an answer that is closest in meaning to a word. Follow strategy when you
don’t know it and choose the right answer.

1)    We read the sentence: Instead, they are deep within the caverns,
quite a distance from the opening to the grotto and any natural
light.

2)    From this sentence, we understand that we are taking about caves and
referring to the fact that the paintings are deep within the cave and far from
the opening of the what? Well, the only thing that makes sense is the opening
to the cave. What else could we be talking about?
 

3)    Each answer is possible, but if you read and understand that the reading
is describing the paintings in the cave, we should guess A. However, every
answer makes sense within the sentence, but we must guess at the most
logical when we don’t know the meaning.

Question 4

This is a vocabulary question and we know this because the question asks us to
identify an answer that is closest in meaning to a word. Follow strategy when you
don’t know it and choose the right answer.

Inhabitant is a word that we must be familiar with. It is a basic word for the
TOEFL. If we do not know this word, then I believe vocabulary may be an issue
and we should spend serious time studying it. Our answer is D.

Question 5

This is a vocabulary question and we know this because the question asks us to
identify an answer that is closest in meaning to a word. Follow strategy when you
don’t know it and choose the right answer.

 
1)    We identify the sentence with perplexing: Three opinions have been
given by the scientific community as to the perplexing origins and
meaning of these paintings.

2)    If we don't know the word, all we do know is that our word serves as an
adjective describing the origins and meaning of the paintings. The content of
the reading can always serve as a clue. The first sentence of the first
paragraph sets the tone and states that these paintings are a source of
mystery. We can assume then that the origins and meaning of the paintings
are a mystery and so mystery is a synonym of perplexing.

3)    We can eliminate A, which means amazing, and B, which means lovable,
and D, which means impossible. Puzzling makes the most sense, so our
answer is C.

Question 6

This is a vocabulary question and we know this because the question asks us to
identify an answer that is closest in meaning to a word. Follow strategy when you
don’t know it and choose the right answer.

Migrations is a basic word that we must know to do fair on the TOEFL. If we do


not know this word, then we must focus seriously on improving our vocabulary.
Migrations means movements, so the answer is B.
 

Question 7

This is a vocabulary question and we know this because the question asks us to
identify an answer that is closest in meaning to a word. Follow strategy when you
don’t know it and choose the right answer.

1)    First, we read the sentence with our word: The fact that in several of
the paintings the animals are portrayed as wounded by arrows or
spears gives particular credence to this theory.

2)    This sentence states essentially that a fact gives credence to the
theory. What could that mean. Let’s search around the sentence to see what
we’re doing to this theory. The sentence after states And the overpainting
is explained by the fact that. . . And implies that we are continuing the
idea from the sentence before. We know what explain means, so we can
assume that credence means something similar to explains, supports,
details, etc.

3)    The answer choices aren’t exactly what we expect and so we guess
towards the answer closest in meaning, which leads us to believability or
substance. We have a 50% chance of getting the question right without
knowing the word, not bad.

 
Our answer is A. Why? Because that’s what the word means.

Question 8

This is a vocabulary question and we know this because the question asks us to
identify an answer that is closest in meaning to a word. Follow strategy when you
don’t know it and choose the right answer.

1)    We read the sentence for clues: And the overpainting is explained
by the fact that the hunters had no use for the paintings after the
hunt itself, and thus felt free to cover them up in preparation for
their next foray into the field.

2)    If we understand the meaning, we know that we’re talking about one
hunt finishing and then the next foray. What could foray mean then?
Another hunt.

3)    If we know the meaning of our answer choices, we know that venture
matches best. The word adventure has venture as its root, so you can
understand that venture has a similar meaning. Hunt couldn’t be
considered a synonym of any of the other answer choices, but it can be a
synonym of an adventure. Our answer is C.

Question 9
 

This is a vocabulary question and we know this because the question asks us to
identify an answer that is closest in meaning to a word. Follow strategy when you
don’t know it and choose the right answer.

1)    We read our sentence: we have long and rigid bodies. In the few cases
that humans are depicted, they are very crudely drawn with long and rigid
bodies.

2)    However we look, it’s tough to find any clue that points to what rigid
besides it being an adjective that can describe a body and it might be similar
in meaning to the word long.

3)    Every answer choice seems possible, so we must know the word in this
case and it is a somewhat basic word. Our answer is C.

*Remember that 4 out of 14 questions will be vocabulary. That’s 28% of your score
that can come easily or with difficulty and guessing. If you’re vocabulary needs
word, start memorizing 7 to 14 words a day.

Question 10

 
This is a vocabulary question and we know this because the question asks us to
identify an answer that is closest in meaning to a word. Follow strategy when you
don’t know it and choose the right answer.

Portray is a basic word that we need to know to expect to do well on the TOEFL.
If we don't know it, we need to study more vocabulary as well. The answer is A.

At this point, you must have memorized the strategy. If you have not, take some
time to do so now.

Final Advice back to top

If you find that you scored 6 out of 10 (or less) vocabulary quesitons, it’s imortant to
start studying vocabulary separately. It will stand in your way of improving. For fast fee
help, there are many vocabulary lists on the internet and you can always build your own
list from these example readings and others. We have also developed a powerful
vocabualry system whose words we’ve taken directly from the most frequently occuring
words from over 60 actual TOEFL readings. It’s a powerful system that can help you as
well. Click here to get an overview it and some further direction on how to improve. If
you have any questions or conerns about it, you can always send us an email
(StudentHelp@NoteFull.com). Whatever you decide to do though, start working on
vocabulary!

 
If you scored 8 out of 10 (or more), studying vocabulary is not a major recommendation.
At this level, it can be a waste of time bceause your vocabulary will build naturally and
it’s more important to practice actual readings than simply memorize words. Of course,
this is unless you know that your vocabulary is an issue, but I don’t recommend studying
it separately unless you have a few hours to study whenever you study and you’re
looking to improve your score as quickly as possible.

  Detail Test

Detail back to top

This question often divides students into two groups: those who will score high and
those who will score low. These questions ask you to identify a fact in the reading and
choose an answer that matches the fact in the reading best. They often begin with
“according to paragraph” and have a WH- question. Here are some examples:

According to paragraph 6, why are limestone and sandstone aquifers not being
refilled?

According to paragraph 1, what is true about the positions of the massive animals
depicted in the paintings?

To score high, you must follow strategy to get to the right information that will lead you
to the right answer.

1)    Read the question to understand what information you are being asked to
find.
2)    Identify the key words in the question to search for in the reading.

3)    Start reading quickly from the beginning of the paragraph in search of the key
words or synonyms of them.

4)    When you find the key word, remind yourself of the question and read
carefully to find the answer for yourself. 60% of the answers will be found in the
sentence with your key words. 30% will require you to read 1 sentence more. 10%
will require you to read 2 sentences more to find the answer.

5)    Once you have identified the answer, check the answer choices and choose the
one that matches your anticipated answer best.

This strategy is the key to success. Do it with full energy and your score will improve. Do
it casually without thinking through steps 1 through 5 and your score will stay the same.
It’s as simple as that.

Reading Practice back to top

Cave Paintings in Lascaux

The Lascaux Grotto, in Southwest France, is a series of narrow cave chambers that was
discovered by children playing in the 1940s. ❒ These caves contained an amazing find
for historians – prehistoric paintings of animals on the walls. ❒ These massive animals
are sometimes portrayed as following one after another in a long line, but on other
occasions, they are sideways on the wall and some even upside down. Paintings are
mostly done in reds, yellow, and brown, and feature such beasts as wild horses, bulls,
reindeer, bison, and mammoths. ❒ Scientists analyzed the substance of the “paint” and
discovered that the colors came from ocher and other iron oxides that had been ground
into a fine powder. ❒ This powder then appears to have been brushed or smeared onto
the cave walls in some cases, while in other cases the artists of these paintings seem to
have blown or sprayed the material onto the rock surface. In these cases, scientists
surmise that tubes made from hollow bone may have been used as tools for the spraying
because hollow bones marked with pigment were discovered close by.

The location of these particular paintings has become a source of mystery to the
scientific community. ❒ Unlike rock paintings found in other parts of the world, these
were not located near the entrance to the cave or completely out in the open. ❒ Instead,
they are deep within the caverns, quite a distance from the opening to the grotto and any
natural light. ❒ In addition, it seems likely that the drawers of these paintings did not
necessarily want them to be seen by others, as inhabitants of caves usually lived
relatively close to the entrances. ❒ The odd location of these particular paintings has led
to various theories as to why so many generations of artists chose to keep their work so
well hidden.

❒ Three opinions have been given by the scientific community as to the perplexing
origins and meaning of these paintings. ❒ The most straightforward of these
explanations is that the paintings are simply a record of the migrations of herds of local
animals. ❒ And since some paintings are drawn directly over others, scientists theorize
that the utility of each painting was over when the migration it depicted had ended. ❒
However, this theory fails to address the puzzling location of these paintings within the
caves, unless it was simply that the cave dwellers chose to commemorate the migrations
with ceremonies held in secret.

 
A second theory holds that these paintings were tied to hunting and they played a vital
role in a special preparation ceremony for the hunters. ❒ The theory states that the
paintings were part of a ritual used to prepare the hunters psychologically for the
challenges they were about to face. ❒ The fact that in several of the paintings the
animals are portrayed as wounded by arrows or spears gives particular credence to this
theory. ❒ And the overpainting is explained by the fact that the hunters had no use for
the paintings after the hunt itself, and thus felt free to cover them up in preparation for
their next foray into the field. ❒

A third opinion on the paintings offers the possibility that the animals portrayed on the
walls may have assumed some sort of mythical meaning to the cave dwellers, either as
ancient ancestors or as protectors of the members of the tribe. Supporters of this theory
point to two different pieces of evidence. First, there are the strange and cryptic
geometric shapes painted near the animals. Second, there is the remarkable absence of
men in any of the paintings. In the few cases that humans are depicted, they are very
crudely drawn with long and rigid bodies. ❒Some of the men are shown lying face down
while others have the heads of birds and other animals. ❒This suggests to advocates of
the theory that the people in the picture might be portrayed in a trance state.
❒However, there is no definitive proof for this theory and it could just as easily be that
these bird-headed men are simply wearing masks for hunting.❒

While these theories continue to compete for primacy in the scientific community, and
there is still heavy debate. It is possible that there will never be a satisfactory answer to
this intriguing question. However, it is clear that the Lascaux paintings represent a great
artistic achievement for the time that will continue to engage researchers, whatever their
significance may be.

 
1.    According to paragraph 1, what material was used to create the colors for the
paintings?

a.    Ocher and other iron oxides

b.    Tubes made from hollow bone

c.    Mammoth bones crushed into a fine powder

d.    Powders smeared onto the walls

2.    According to paragraph 1, what is true about the positions of the massive animals
depicted in the paintings?

a.    They are uniform throughout the caves.

b.    They are sometimes in seemingly random positions.

c.    The majority of the animals are upside down and sideways.

d.    Their position varied depending on the size of the painting.

3.    According to paragraph 2, what is true about the locations of the paintings in
Lascaux?

a.    They are of no particular significance.

b.    Their close proximity to the mouth of the caves has contributed to the mystery
in the scientific community.

c.    The lack of light deep in the cave has made deciphering the meaning difficult.

d.    The unusual location of the paintings has led to varying theories.
 

4.    What is stated in paragraph 2 about the natural light in Lascaux?

a.    It may not have reached the location of the paintings.

b.    It reached only a short distance from the cave entrance.

c.    It reached deep within the cave, aiding the artists.

d.    It made painting in Lascaux more stylized than in other caves.

5.    According to paragraph 3, what is one explanation for the Lascaux paintings?

a.    They were probably used to predict migration patterns.

b.    They may have documented the movement of groups of animals.

c.    The herds depicted in the drawings have puzzled scientists.

d.    They usually depicted the ends of the migratory season.

6.    What is stated in paragraph 3 about the theory mentioned?

a.    It is widely accepted because of its straightforward nature.

b.    It is put forward to explain migratory habits of prehistoric animals.

c.    It does not address the issue of overpainting.

d.    It does not completely account for all the facts.

 
7.    According to the theory in paragraph 4, what was the purpose of the paintings?

a.    To help warriors prepare to battle neighboring tribes

b.    To document past hunting triumphs of the tribe

c.    To make hunters mentally ready for the challenge of the hunt

d.    To show compassion for animals wounded by spears or arrows

8.    According to the theory in paragraph 4, why were the paintings covered by later
paintings?

a.    They were no longer needed after the hunt.

b.    The artists were constantly improving their craft.

c.    Overpainting was a part of the ritualistic preparations.

d.    Overpainting was part of the next foray into the field.

9.    According to the theory in paragraph 5, what may the animals in the paintings have
represented to the cave’s inhabitants?

a.    past members of the tribe

b.    the artists themselves

c.    members of the hunting party

d.    the lack of men in the tribe

 
10.    According to paragraph 5, what can be said about the geometric shapes near the
pictures of the animals?

a.    They represent a more abstract depiction of the animals.

b.    They represented past lives of the artists.

c.    They may have been tied to the religious rituals of the tribe.

d.    They may help to explain why animals were portrayed in the paintings.

11.    According to paragraph 6, what is true about the controversy surrounding the cave
paintings at Lascaux?

a.    The most recent theories will be proven in the scientific community.

b.    The question may never be answered definitively.

c.    The theories will eventual agree on the basics.

d.    The theories are intriguing because of their significance.

12.    According to paragraph 6, what can be said about the cave paintings at Lascaux?

a.    They have since been supplanted by more intriguing discoveries.

b.    Their significance is diminished by the controversy surrounding them.

c.    Researchers are closing in on a solution to their mystery.

d.    They were a major accomplishment for the art of that period.

 
Power Review back to top

Remember that it’s all about strategy. If you don’t have the steps memorized for finding
a detail question, you are losing points. There’s no more powerful way I can say it than
that. If it’s not memorized yet, please memorize it now. Remember that this is for your
score.

1)    Read the question to understand what information you are being asked to
find.

2)    Identify the key words in the question to search for in the reading.

3)    Start reading quickly from the beginning of the paragraph in search of the key
words or synonyms of them.

4)    When you find the key word, remind yourself of the question and read
carefully to find the answer for yourself. 60% of the answers will be found in the
sentence with your key words. 30% will require you to read 1 sentence more. 10%
will require you to read 2 sentences more to find the answer.

5)    Once you have identified the answer, check the answer choices and choose the
one that matches your anticipated answer best.

Question 1

This is a detail question. We know this because it asks us to identify a fact in the
reading. We remember our strategy and follow it exactly.
 

1)    First, we want to understand the question and note the specific keywords
within that question: what material was used to create the colors for
the paintings? Before we start to look at the reading for the answers, we
want to know what we’re looking for. This question asked what. That means
that were looking for nouns--things. When we look at the answer choices, we
actually see a series of nouns. This therefore should be considered an easy
question.

2)    Our key words: materials, create, colors, paintings.

3)    We start reading and in the fourth sentence, we find two of our key
words (and synonyms of them): paintings are mostly done in reds,
yellow, and brown, and feature such beasts as wild horses, bulls, reindeer,
bison, and mammoths.

4)    Remember that we want to know what those colors in the paintings are
made of. We read the sentence carefully and don’t find the information
there, so what do we do? We keep reading knowing that the next sentence
has the answer about 30% of the time: Scientists analyzed the
substance of the “paint” and discovered that the colors came from
ocher and other iron oxides that had been ground into a fine
powder. For us, the next sentence gives us our answer. Came from
identifies the material used to create the colors: ocher and other iron
oxides.

5)    We check our answers and find that there’s only one possibility: A.

This is how we want to answer the question, systematically. Also, always work to
find the answer in the reading before you look at the answer choices. This alone
will improve your score because you will not be tricked by wrong answers.
 

Question 2

This is a detail question. We know this because it asks us to identify a fact in the
reading. We remember our strategy and follow it exactly.

1)    First, we identify our question: what is true about the positions of
the massive animals depicted in the paintings. We are being asked for
any true statement about the position of the animals.

2)    Our key words: positions, massive animals, depicted, paintings.

3)    We go back to our reading and search for our keywords. We see a
synonym of position in the 3rd sentence and find many of our key words
there: These massive animals are sometimes portrayed as following one
after another in a long line, but on other occasions, they are sideways
on the wall and some even upside down.

4)    We identify that sentence as the key sentence to provide us with our
answer. What is true about the positions from this sentence? From this
sentence, we find that the animals are found in one of three positions: one
after another, sideways, and upside down.

5)    We check our answer choices. A is wrong because uniform means the
same and our sentence states that they are not the same position. B is not a
perfect answer but it looks good because random means there is no pattern
or order; there is variety. C is wrong because there is no mention about
which position is most common in the paintings. Majority makes C wrong.
D is wrong because there is no mention of size of the painting in our
sentence. Our answer is, therefore, B.
 

Question 3

This is a detail question. We know this because it asks us to identify a fact in the
reading. We remember our strategy and follow it exactly.

1)    First, we identify our question: what is true about the locations of
the paintings in Lascaux. Again, we must simply learn a fact from the
reading about the location of the paintings.

2)    Our key words: locations, paintings, Lascaux.

3)    We see location and our other key words everywhere in the paragraph.

4)    Therefore know that this is a tough question because we must read each
sentence carefully and search for our answer actively going back and forth
from our reading and answer choices.

5)    We read the first sentence and learn that A is incorrect because the first
sentence tells us that they're a source of mystery to the community. As a
result, scientists are studying them and so they are of significance. B is
incorrect because we learn in the second sentence that the paintings are far
and not close to the entrance of the cave. C references keywords in the
paragraph and looks correct until we get to the last half and pay attention to
the meaning. We do not have a cause-and-effect relationship established
between the fact that there is little light in the cave and the fact that the
meaning of the painting is difficult to understand in the reading, so C is
wrong. D is correct because of the last sentence in the paragraph: The odd
location of these particular paintings has led to various theories
as to why so many generations of artists chose to keep their work
so well hidden. It states the same information as in answer choice D.

This is a tough question because we have to read through the whole paragraph to
get to the answer. You should expect to see at least one detail question like this on
the exam.

Question 4

This is a detail question. We know this because it asks us to identify a fact in the
reading. We remember our strategy and follow it exactly.

1)    First, we identify our question: what is stated about the natural
light in Lascaux. Again, we’re just looking for a true fact about the
natural light.

2)    Our key words: natural light, Lascaux.

3)    We find our key words in the third sentence: Instead, they are deep
within the caverns, quite a distance from the opening to the grotto and any
natural light. We read the sentence carefully.

4)    We read the sentence and learn that they, which references the
paintings, are very far from natural light. This is a fact and a possible answer.

5)    We check our answer choices. A looks good. This answer appears more
as an inference than a detail, which sometimes happens, but it can be
inferred from our sentence: since natural light is far from the paintings, it
makes sense to say that natural light did not reach, or touch, the paintings;
how could it? B is not supported. We only know that the paintings were far
from natural light, we do not know how far the natural light traveled; it could
have been short or long, all we know was that the light was far. C goes
against the sentence; how could the natural light be far from the painting
and reach it. A tennis ball cannot be far from you and close to you at the
same time. D is 100% wrong. We read nothing about style. Our answer is A.

Question 5

This is a detail question. We know this because it asks us to identify a fact in the
reading. We remember our strategy and follow it exactly.

1)    First, we identify our question: what is one explanation for the
Lascaux paintings. This question seems very general, but we know we’re
looking for an explanation of the paintings. We should remember that the
passage states that 3 will be discussed.

2)    Our key words: explanation, Lascaux paintings.

3)    We read paragraph 3 looking for an explanation of the paintings. The
second sentence of the paragraph has our key words: The most
straightforward of these explanations is that the paintings are simply a
record of the migrations of herds of local animals.

4)    This sentence contains our answer: . . . explanations is that the


paintings are simply a record of the migrations of herds of local
animals. We know our answer: record migrations.
5)    We check our answers. A contains the word migration, but it creates a
false meaning. Our sentence states that the paintings were used as records
and not for predictions. That is not the same meaning and A is wrong. B
uses documented as a synonym of record and movement of animals is a
synonym of migrations. B looks very good. We continue because we must
always check every answer just in case we made a mistake. C is incorrect
because we read the word herd in the reading, but we never see that the
herds themselves puzzled or confused the scientists. D references
migration, but again, it creates a false meaning. The reading states that the
paintings record the migration, not that they record the ends of
migratory seasons. B is our answer.

Some of you may argue that D is possible. Remember that we must choose the best
answer out of the 4 possible. Sometimes we really can argue for 2 answers, but this
is a TOEFL test and there is one clear answer that is more connected to the
reading than another. B is an accurate restatement of the sentence and therefore
should not be argued against.

Question 6

This is a detail question. We know this because it asks us to identify a fact in the
reading. We remember our strategy and follow it exactly.

1)    First, we identify our question: what is stated about the theory
mentioned. Here, we’re simply looking for a fact that comes directly from
the reading.
2)    Our key word: theory.

3)    We go back to the 3rd paragraph.

4)    The whole paragraph is about the theory. We know therefore that we
must go back and forth between the reading and the answer choices.

5)    The first sentence states that the theory is about the origin and meaning
of the painting. No answer choice states that. We continue reading. Next, we
learn: The most straightforward of these explanations is that the
paintings are simply a record of the migrations of herds of local
animals. A uses the keyword straightforward but it creates a false
meaning. We have no reference to the fact that this is widely accepted. A is
wrong so far. B is incorrect. We have our keyword migratory, but our
sentence does not talk about habits. Also, put forward in answer choice B
means proposed or offered. The theory is meant to explain the origin and
meaning of the paintings; it is not meant to explain the migratory habits.
B is wrong. C, we have not read about. D we have not read about. However,
if we’re running out of time, we should see that C or D are the likely answers
and now we have a good chance of guessing correctly and improving our
score. We continue reading to determine whether it’s C or D. The next
sentence states: And since some paintings are drawn directly over
others, scientists theorize that the utility of each painting was
over when the migration it depicted had ended. This does not help
us. We continue. The next sentence states: However, this theory fails to
address the puzzling location of these paintings within the caves,
unless it was simply that the cave dwellers chose to commemorate
the migrations with ceremonies held in secret. Immediately, we
should recognize D as the answer because it states that the theory does not
completely account for all the facts; the one fact it doesn't account for
is the puzzling location of these paintings. C we should not have chosen
because overpainting is not mentioned at all in this paragraph. If we did,
then we must remember our strategy.
 

Question 7

This is a detail question. We know this because it asks us to identify a fact in the
reading. We remember our strategy and follow it exactly.

1)    First, we identify our question: what was the purpose of the
paintings. We are looking for the purpose of something and that is why all
of the answers start with infinitives, which indicate purpose.

2)    Our key words: paintings.

3)    This is a very general keyword and we see it everywhere in the


paragraph.

4)    We immediately understand that we are going to check the reading with
the answer choices one by one as we read each sentence.

5)    The first sentence tells us that a theory states that the purpose of the
paintings was connected with hunting. This is a possible answer and so we
look at our answer choices. A doesn't make sense because they're preparing
to hunt and not to battle tribes. B doesn't make sense because document
hunting triumphs is not mentioned. C looks good because we know that
the purpose of the paintings was connected with hunting.  D is not a good
answer because it talks about compassion for animals, which goes against
the idea described in the first sentence of hunting them, so D is wrong
because it goes against the passage. If we’re stressed for time, we choose C
and move on; however, we didn’t read about prepare hunters mentally yet. If
we keep reading (which we always should when the answer isn’t 100% clear
from our sentence), the next sentence tells us that the paintings were to
prepare the hunters for the hunt psychologically, which is a synonym of
mentally in choice C. C is our answer.

Question 8

This is a detail question. We know this because it asks us to identify a fact in the
reading. We remember our strategy and follow it exactly.

1)    First, we identify our question: why were the paintings covered by
later paintings.

2)    Our key words: paintings covered by later paintings.

3)    We continue reading paragraph 4 looking for our keywords. We find our
key words in the last sentence: And the overpainting is explained by the
fact that the hunters had no use for the paintings after the hunt itself, and
thus felt free to cover them up in preparation for their next foray into the
field.

4)    We read carefully trying to learn why the paintings were over painted.
We see is explained by the fact that and know that the words that follow
that phrase will tell us why: the hunters had no use for the paintings
after the hunt itself. That's it; that’s our reason.

5)    We look for the answer and find that A restates this information
perfectly. Of course, we still check the other answer choices in case we made
a mistake. B is wrong because we never read about improving their craft.
C is wrong because we don't read that overpainting itself was part of the
ritual in our sentence; overpainting occurred after the ritual and the hunt. D
is wrong because overpainting was not a part of the next foray, which means
hunt. Don't be tricked by this answer simply because it has a key word from
the sentence. Always check the meaning. Our answer is A.

Question 9

This is a detail question. We know this because it asks us to identify a fact in the
reading. We remember our strategy and follow it exactly.

1)    First, we identify our question: what may the animals in the
paintings have represented to the cave inhabitants.

2)    Our key words: animals, paintings, represent, cave inhabitants.

3)    We go to paragraph 5 and look for our keywords or synonyms of them to
find the sentence that likely contains our answer. In the first sentence: A
third opinion on the paintings offers the possibility that the animals
portrayed on the walls may have assumed some sort of mythical
meaning to the cave dwellers, either as ancient ancestors or as protectors
of the members of the tribe. We have animals portrayed on the walls,
which is a synonym of paintings. We also have the words may have
assumed, which means represented. It sounds like we have a sentence
that contains our answer.

4)    It states: . . . may have assumed some sort of mythical meaning
to the cave dweller's, either as ancient ancestors or as protectors
of the members of the tribe. So, the animals could have had a mythical
meaning of an ancient ancestor or a protector of the tribe. We have formed
our answer.
5)    We look at our answer choices. A looks good because past members of
the tribe is a synonym of ancient ancestors. B doesn't look good because
the artists of the paintings could not have been the ancient ancestors or
protectors. For the same reason, we eliminate C. Members of the hunting
party are not described as the ancient ancestors or protectors; they are
just hunters. D is also a bad answer because there is no reference to a lack
of men. As a result, with the right strategy, we find the answer easily: A.

Question 10

This is a detail question. We know this because it asks us to identify a fact in the
reading. We remember our strategy and follow it exactly.

1)    First, we identify our question: what can be said about the
geometric shapes near the pictures of the animals. Because we have
can be said, this could be an inference question. Because it asks for a
possibility instead of a fact. It’s a bit trick, so be prepared for a tougher
answer though you may not need to be.

2)    Our key words: geometric shapes, pictures, animals.

3)    We go to paragraph 5 and scan through the paragraph quickly for our
keywords. As we read we see our keyword in the 3rd sentence: First, there
are the strange and cryptic geometric shapes painted near the animals.

4)    The adverb first tells us that this sentence is part of a series; it is the
first of something. We focus on the sentence before. It states: supporters
of this theory point to two different pieces of evidence. Our sentence
therefore is evidence that supports a theory. What theory? Since we have the
word this in the sentence we must continue reading the sentence before: A
third opinion on the paintings offers the possibility that the
animals portrayed on the walls may have assumed some sort of
mythical meaning as ancient ancestors or protectors. So, the
geometric shapes support that. We have our answer. If we read carefully as
we go we wouldn't have to do this much work in going backwards because we
would already understand this. Always works to improve reading
comprehension. But, strategy will still save us.

5)    Let's look at our answer choices. A contains a key word that we didn’t
read about: abstract depiction. So, A is wrong. B is wrong because they
refers to the geometric shapes in the question and those shapes did not
represent past lives. C has no bases of support. We did not read that the
geometric shapes were tied to religious rituals. D states that the shapes
may help to explain why animals were portrayed in the paintings.
This is a tricky answer choice because it makes us think of bit more deeply.
What are the three theories focusing on? The meaning of the animals in the
paintings. So, the theory that we read is meant to explain the meaning of the
animals in the paintings. Why were they there? As you can see, D matches
are meaning best.

Question 11

This is a detail question. We know this because it asks us to identify a fact in the
reading. We remember our strategy and follow it exactly.

1)    First, we identify our question: what is true about the controversy
surrounding the cave paintings at Lascaux.
2)    Our key words: controversy surrounding, paintings, Lascaux.

3)    We read paragraph 6 and find our keyword in the first sentence: while
these theories continue to compete for primacy in the scientific
community, and there is still heavy debate. Heavy debates is a synonym
of controversy. We read that sentence carefully

4)    The sentence tells us that neither of the theories is number one but is
competing for number one (competing for primacy). We look at our
answer choices to see if this gives us an answer. And no answer states this
information. We keep reading: it is possible that there will never be a
satisfactory answer to this intriguing question. This sentence states
that there may never be a satisfactory answer.

5)    We check our answers and find that A is wrong because it opposes the
idea mentioned. Will be proven means that there will be an answer. B
states our sentence exactly and is a great answer. C references information
that you never read about: agree on the basics. D has nothing to do with
the information that we read: intriguing because of their significance.
B is our answer.

Question 12

This is a detail question. We know this because it asks us to identify a fact in the
reading. We remember our strategy and follow it exactly.

1)    First, we identify our question: what can be said about the cave
paintings at Lascaux. We are searching for a description of the cave
paintings. Something about them.
2)    Our key words: cave paintings, Lascaux.

3)    We find our key words in the last sentence: however, it is clear that the
Lascaux paintings represent a great artistic achievement for the time that
will continue to engage researchers, whatever their significance may be.

4)    We learned that they are a major artistic achievement.

5)    We check our answer choices. A is wrong because it states that they have
been supplanted (which means replaced) by more intriguing
discoveries. B is wrong because we don't read about diminished
significance. C is wrong because it goes against the reading and is not
mentioned in our sentence. D is perfect because it restates our information
well. This is an easy detail question.

Final Advice back to top

  

This is the most common question on the TOEFL and therefore, it needs to be mastered.
Make sure that if you scored less than 8 out of 12 on this that you review it at least twice
more. Can you repeat each step from memory? Do you go through the question thinking
about: step 1, then 2, then 3? Or do you just answer it naturally? The point is not to take
out your brain as you answer but to give you an intelligent strategy to supplement, to
add to, your thought process. Don't follow strategy without thinking. Remember that.
Follow strategy to help you get to the answer more quickly and efficiently.

Play with the method and make it your own. Good luck and review is the key to get it all
comfortable and usable on the test.

 Negative Detail Test


Negative Detail Strategy (a subset of detail questions) back to top

These questions ask you to identify an answer that is NOT mentioned in the paragraph
or that is NOT correct according to the reading. They always possess within the question
the words NOT or EXCEPT capitalized and in bold. Here is an example:

According to paragraph 5, all of the following are true about the men in the cave
paintings EXCEPT:

To score high, you must follow strategy to get to the right information that will lead you
to the right answer. In these questions, it’s best to go back and forth from the reading to
the answer choices.

1)    Read the question to understand what information you are being asked to
find.

2)    Identify the key words in the question to search for in the reading.

3)    Start reading quickly from the beginning of the paragraph in search of the key
words or synonyms of them.

4)    When you find the key word, read the sentence and check the answer choices
to determine which you can eliminate. Continue to do this for each answer choice
until you can determine that one is wrong or not mentioned and choose that
answer.

 
This strategy is the key to success. Do it with full energy and your score will improve. Do
it casually without thinking through steps 1 through 4 and your score will stay the same.

Reading Practice back to top

Cave Paintings in Lascaux

The Lascaux Grotto, in Southwest France, is a series of narrow cave chambers that was
discovered by children playing in the 1940s. ❒ These caves contained an amazing find
for historians – prehistoric paintings of animals on the walls. ❒ These massive animals
are sometimes portrayed as following one after another in a long line, but on other
occasions, they are sideways on the wall and some even upside down. Paintings are
mostly done in reds, yellow, and brown, and feature such beasts as wild horses, bulls,
reindeer, bison, and mammoths. ❒ Scientists analyzed the substance of the “paint” and
discovered that the colors came from ocher and other iron oxides that had been ground
into a fine powder. ❒ This powder then appears to have been brushed or smeared onto
the cave walls in some cases, while in other cases the artists of these paintings seem to
have blown or sprayed the material onto the rock surface. In these cases, scientists
surmise that tubes made from hollow bone may have been used as tools for the spraying
because hollow bones marked with pigment were discovered close by.

The location of these particular paintings has become a source of mystery to the
scientific community. ❒ Unlike rock paintings found in other parts of the world, these
were not located near the entrance to the cave or completely out in the open. ❒ Instead,
they are deep within the caverns, quite a distance from the opening to the grotto and any
natural light. ❒ In addition, it seems likely that the drawers of these paintings did not
necessarily want them to be seen by others, as inhabitants of caves usually lived
relatively close to the entrances. ❒ The odd location of these particular paintings has led
to various theories as to why so many generations of artists chose to keep their work so
well hidden.

❒ Three opinions have been given by the scientific community as to the perplexing
origins and meaning of these paintings. ❒ The most straightforward of these
explanations is that the paintings are simply a record of the migrations of herds of local
animals. ❒ And since some paintings are drawn directly over others, scientists theorize
that the utility of each painting was over when the migration it depicted had ended. ❒
However, this theory fails to address the puzzling location of these paintings within the
caves, unless it was simply that the cave dwellers chose to commemorate the migrations
with ceremonies held in secret.

A second theory holds that these paintings were tied to hunting and they played a vital
role in a special preparation ceremony for the hunters. ❒ The theory states that the
paintings were part of a ritual used to prepare the hunters psychologically for the
challenges they were about to face. ❒ The fact that in several of the paintings the
animals are portrayed as wounded by arrows or spears gives particular credence to this
theory. ❒ And the overpainting is explained by the fact that the hunters had no use for
the paintings after the hunt itself, and thus felt free to cover them up in preparation for
their next foray into the field. ❒

A third opinion on the paintings offers the possibility that the animals portrayed on the
walls may have assumed some sort of mythical meaning to the cave dwellers, either as
ancient ancestors or as protectors of the members of the tribe. Supporters of this theory
point to two different pieces of evidence. First, there are the strange and cryptic
geometric shapes painted near the animals. Second, there is the remarkable absence of
men in any of the paintings. In the few cases that humans are depicted, they are very
crudely drawn with long and rigid bodies. ❒Some of the men are shown lying face down
while others have the heads of birds and other animals. ❒This suggests to advocates of
the theory that the people in the picture might be portrayed in a trance state.
❒However, there is no definitive proof for this theory and it could just as easily be that
these bird-headed men are simply wearing masks for hunting.❒

While these theories continue to compete for primacy in the scientific community, and
there is still heavy debate. It is possible that there will never be a satisfactory answer to
this intriguing question. However, it is clear that the Lascaux paintings represent a great
artistic achievement for the time that will continue to engage researchers, whatever their
significance may be.

1)    According to paragraph 1, all of the following creatures are mentioned as being
depicted on the cave walls EXCEPT:

a)    Mammoths

b)    Ochers

c)    Reindeer

d)    Bison

2)    In paragraph 2, the author mentions all of the following about the location of the
paintings EXCEPT:

a)    They were located far from the mouth of the cave.

b)    They were in a secluded part of the cave, unlike other cave paintings.
c)    The location deep within the cave may have been an attempt to hide the
paintings from others.

d)    Their location was consistent with other findings of rock paintings.

3)    According to paragraph 5, all of the following are true about the men in the cave
paintings EXCEPT:

a)    They may have been portrayed lying down in a trance position.

b)    They were drawn with different colors than the animals.

c)    They sometimes were drawn with the heads of other animals.

d)    Their bodies were long and rigid.

Power Review back to top

Take a moment to review and memorize the strategy if you haven’t already.

1)    Read the question to understand what information you are being asked to
find.

2)    Identify the key words in the question to search for in the reading.

3)    Start reading quickly from the beginning of the paragraph in search of the key
words or synonyms of them.
4)    When you find the key word, read the sentence and check the answer choices
to determine which you can eliminate. Continue to do this for each answer choice
until you can determine that one is wrong or not mentioned and choose that
answer.

Question 1

This is a negative detail question. We know this because it has the word EXCEPT
or NOT. We follow our strategy carefully.

1)    First, we identify our question: all of the following creatures are
mentioned as being depicted on the cave walls EXCEPT. We know
that we’re looking for creatures painted on the walls.

2)    Our key words: creatures, depicted, cave walls.

3)    We find synonyms of creatures in the fourth sentence: Paintings are
mostly done in reds, yellow, and brown, and feature such beasts as wild
horses, bulls, reindeer, bison, and mammoths.

4)    Now, we’re looking for the items that were not on the cave walls. The
answer choices are nouns, so we just have to find what's not mentioned. The
reading mentioned wild horses, polls, reindeer, bison, and
mammoths. A, C, and D are mentioned and therefore our answer is B.
Ocher is actually the material that is used to create paint. This was an easy
negative detail question.

 
Question 2

This is a negative detail question. We know this because it has the word EXCEPT
or NOT. We follow our strategy carefully.

1)    First, we identify our question: the author mentions all of the
following about the location of the paintings EXCEPT. We are
simply being asked to find facts about the location of the paintings and
determine what is not mentioned.

2)    Our key words: location, paintings.

3)    The first sentence has our key word: The location of these particular
paintings has become a source of mystery to the scientific community.

4)    This means we must pay careful attention from the very beginning. We
go back and forth between the reading and the answer choices. We first learn
that the location is a source of mystery, but that isn't an answer choice. We
then learn in the next sentence that the paintings were not located near the
entrance. Next, we learn that they were deep inside. We can now, from the
information in the third sentence, eliminate A and B. Then, we learn that the
painters may not have wanted the paintings to be seen by others. This
eliminates answer choice C. We find, therefore, that the answer is D.

*We could have guessed that D was the answer sooner in our analysis because the
second sentence stated that unlike rock paintings found in other parts of
the world. D stated that the location of the paintings in Lascaux was consistent
with other findings, which directly opposes the reading and makes D instantly
incorrect. Remember that there are two ways to arrive at the right answer for
negative detail questions: eliminating what was mentioned or choosing what is
incorrect from the reading.

Question 3

This is a negative detail question. We know this because it has the word EXCEPT
or NOT. We follow our strategy carefully.

1)    First, we identify our question: all of the following are true about
the men in the cave paintings EXCEPT. We know we are being asked to
find information about the men in the paintings.

2)    Our key words: men, cave paintings.

3)    We first find our key words in the fourth sentence: Second, there is the
remarkable absence of men in any of the paintings. That tells us that men
were not in the paintings (clearly). We look at the answer choices and don't
see one that states this. We keep reading. The next sentence states that
humans were drawn with long and rigid bodies. That points to D and
therefore eliminates it. We continue. The next sentence states that men were
shown lying face down and some had heads of birds or other animals. This
points to A but we don't read about trance position, so A stays as a
possible answer. C is eliminated. We continue. The next sentence states this
suggests a trance state. This refers to the sentence before, which
describes the positions of the men (lying face down), which eliminates A.

4)    B by process of elimination is our answer. We don't read anything about


different colors in the paragraph.
  

Final Advice back to top

This question is often a matter of time and focus. Make sure that you pay attention
because students often get these questions wrong when they lose focus. Just remember
that these questions need your full attention (as all questions do) and reminding
yourself of that should help you avoid careless errors.

  Inference Test

Inference Strategy back to top

Inference questions are difficult. They usually contain the words infer, imply, or suggest
in the question. Here are a few examples to know when you read one.

What can be inferred in paragraph 7 about shallow aquifers?

What is implied in the theory mentioned in paragraph 3 about the fact that some
paintings in Lascaux are drawn directly over others?

The strategy is the exact same as that for detail questions except that when you read the
sentence with your key word make a jump from it with respect to content. For example,
if you read: 30% of apples are green. What does that mean? Well, a detail question just
asks how many apples are green: 30%. An inference question asks what can you infer
about apples: 70% of apples aren’t green. Now, you didn’t read that but you can infer it
from the information, right. That’s the idea behind an inference question and that’s why
they arguably cause the most trouble to students.

1)    Read the question to understand what information you are being asked to
find.

2)    Identify the key words in the question to search for in the reading.

3)    Start reading quickly from the beginning of the paragraph in search of the key
words or synonyms of them.

4)    When you find the key word, remind yourself of the question and read
carefully to find the answer for yourself. Ask yourself, what does this imply that
must be true? 60% of the answers will be found in the sentence with your key
words. 30% will require you to read 1 sentence more. 10% will require you to read
2 sentences more to find the answer.

5)    Once you have identified the answer, check the answer choices and choose the
one that matches your anticipated answer best.

6)    If you don’t find the answer right away, keep asking yourself: what must be
true from this sentence that isn’t stated directly. (Remember the apple example.)

Reading Practice back to top

Cave Paintings in Lascaux

 
The Lascaux Grotto, in Southwest France, is a series of narrow cave chambers that was
discovered by children playing in the 1940s. ❒ These caves contained an amazing find
for historians – prehistoric paintings of animals on the walls. ❒ These massive animals
are sometimes portrayed as following one after another in a long line, but on other
occasions, they are sideways on the wall and some even upside down. Paintings are
mostly done in reds, yellow, and brown, and feature such beasts as wild horses, bulls,
reindeer, bison, and mammoths. ❒ Scientists analyzed the substance of the “paint” and
discovered that the colors came from ocher and other iron oxides that had been ground
into a fine powder. ❒ This powder then appears to have been brushed or smeared onto
the cave walls in some cases, while in other cases the artists of these paintings seem to
have blown or sprayed the material onto the rock surface. In these cases, scientists
surmise that tubes made from hollow bone may have been used as tools for the spraying
because hollow bones marked with pigment were discovered close by.

The location of these particular paintings has become a source of mystery to the
scientific community. ❒ Unlike rock paintings found in other parts of the world, these
were not located near the entrance to the cave or completely out in the open. ❒ Instead,
they are deep within the caverns, quite a distance from the opening to the grotto and any
natural light. ❒ In addition, it seems likely that the drawers of these paintings did not
necessarily want them to be seen by others, as inhabitants of caves usually lived
relatively close to the entrances. ❒ The odd location of these particular paintings has led
to various theories as to why so many generations of artists chose to keep their work so
well hidden.

❒ Three opinions have been given by the scientific community as to the perplexing
origins and meaning of these paintings. ❒ The most straightforward of these
explanations is that the paintings are simply a record of the migrations of herds of local
animals. ❒ And since some paintings are drawn directly over others, scientists theorize
that the utility of each painting was over when the migration it depicted had ended. ❒
However, this theory fails to address the puzzling location of these paintings within the
caves, unless it was simply that the cave dwellers chose to commemorate the migrations
with ceremonies held in secret.

A second theory holds that these paintings were tied to hunting and they played a vital
role in a special preparation ceremony for the hunters. ❒ The theory states that the
paintings were part of a ritual used to prepare the hunters psychologically for the
challenges they were about to face. ❒ The fact that in several of the paintings the
animals are portrayed as wounded by arrows or spears gives particular credence to this
theory. ❒ And the overpainting is explained by the fact that the hunters had no use for
the paintings after the hunt itself, and thus felt free to cover them up in preparation for
their next foray into the field. ❒

A third opinion on the paintings offers the possibility that the animals portrayed on the
walls may have assumed some sort of mythical meaning to the cave dwellers, either as
ancient ancestors or as protectors of the members of the tribe. Supporters of this theory
point to two different pieces of evidence. First, there are the strange and cryptic
geometric shapes painted near the animals. Second, there is the remarkable absence of
men in any of the paintings. In the few cases that humans are depicted, they are very
crudely drawn with long and rigid bodies. ❒Some of the men are shown lying face down
while others have the heads of birds and other animals. ❒This suggests to advocates of
the theory that the people in the picture might be portrayed in a trance state.
❒However, there is no definitive proof for this theory and it could just as easily be that
these bird-headed men are simply wearing masks for hunting.❒

While these theories continue to compete for primacy in the scientific community, and
there is still heavy debate. It is possible that there will never be a satisfactory answer to
this intriguing question. However, it is clear that the Lascaux paintings represent a great
artistic achievement for the time that will continue to engage researchers, whatever their
significance may be.

1)    According to paragraph 1, what can be inferred from the fact that artists seem to
have blown material onto the surface of their paintings?

a)    They did not stay in the Lascaux Grotto for a long time.

b)    They may have had the capacity to craft rudimentary tools.

c)    They only used one method to apply the paint to the walls.

d)    They feared the animals depicted in the paintings.

2)    In paragraph 2, what can be inferred about the artists of most cave paintings other
than those in Lascaux?

a)    They did not want their work to be seen.

b)    They tended to work in natural light.

c)    They were used to working without tools.

d)    They kept their living space and painting space separate.

3)    What is implied in the theory mentioned in paragraph 3 about the fact that some
paintings in Lascaux are drawn directly over others?

a)    It may not have happened if the paintings had been needed.

b)    It may have occurred because of the paintings’ puzzling location.
c)    It was a straightforward record of the migrations of herds.

d)    It might have indicated that the migrations varied from year to year.

4)    According to paragraph 4, what can be inferred from the presence of spears and
arrows in some of the paintings?

a)    It is a sign that weaponry was rather advanced in the tribe.

b)    It is an indication that the animals depicted were prey to the tribes.

c)    It explains why the hunters had no use for the paintings prior to the hunt.

d)    It shows the centrality of the ritual in the life of the tribe.

5)    What is implied in paragraph 4 about the hunting done by the cave dwellers if the
paintings were in fact part of a ritual?

a)    It was not sustainable season to season.

b)    Weapons more advanced than arrows and spears were necessary to be
successful

c)    It played a vital role in the special preparation ceremony.

d)    It required the hunters to be motivated and focused.

6)    What does the information in paragraph 5 about the possible mythical meaning of
the paintings to cave dwellers imply about the cave dwellers?

a)    They may have revered their ancestors.


b)    Their distant ancestors may not have been members of the tribe.

c)    They believed that the trance was a way to gain power in the tribe.

d)    They took on a mythical importance in the drawings.

7)    What can be inferred in paragraph 6 about the scientific community’s reaction to
the paintings?

a)    They are still interesting to scientists in spite of the unresolved controversy.

b)    Over time, interest in them has waned because of their unknown origins.

c)    They are a major area of current scientific research.

d)    The scientific community is encouraged by the mystery surrounding them.

Power Review back to top

Remember our strategy. If you don’t, take the time to memorize it now and let’s review.

1)    Read the question to understand what information you are being asked to
find.

2)    Identify the key words in the question to search for in the reading.

3)    Start reading quickly from the beginning of the paragraph in search of the key
words or synonyms of them.
4)    When you find the key word, remind yourself of the question and read
carefully to find the answer for yourself. Ask yourself, what does this imply that
must be true? 60% of the answers will be found in the sentence with your key
words. 30% will require you to read 1 sentence more. 10% will require you to read
2 sentences more to find the answer.

5)    Once you have identified the answer, check the answer choices and choose the
one that matches your anticipated answer best.

6)    If you don’t find the answer right away, keep asking yourself: what must be
true from this sentence that isn’t stated directly. (Remember the apple example.)

Question 1

This is an inference question. We know this because it asks us what the reading
infers, implies, or suggests.

1)    First, we read and understand our question: what can be inferred
from the fact that artists seem to have blown material onto the
surface of their paintings. We want to know what does the fact that the
artists did this act tell us.

2)    Our key words: artists, blown material, surface, painting.

3)    We find our key words in the 2nd to last sentence of the paragraph: This
powder then appears to have been brushed or smeared onto the cave walls in
some cases, while in other cases the artists of these paintings seem to have
blown or sprayed the material onto the rock surface.
4)    We read that sentence carefully. When we read the sentence, we just get
a fact: this was one method in other cases that was used to paint the
surface. What can we infer about this fact? What does the sentence tell us
indirectly? We can infer that this was one of many methods that the artists
may have used to get material onto the cave walls since they may have also
brushed or smeared paint onto the cave walls.

5)    We look at the answer choices to see if this is an answer choice. We can
only find that C is wrong because of what we understood; there is more than
one method. Now, if we guess, we are more likely to guess the right answer.

6)    Since we couldn't choose an answer from what we understood, let's read
the next sentence and see if we learn more about what blowing material
might infer: In these cases, scientists surmise that tubes made from
hollow bone may have been used as tools for the spraying because
hollow bones marked with pigment were discovered close by. In
these cases, functions as a pronoun referring back to the sentence before.
So, we see that our two sentences are very connected. To make things easier
in our head, let's replace the pronouns with the actual nouns. In these
paintings (seem to have blown or spray the material onto the rock
surface), scientists surmise that tubes made from hollow bone
may have been used as tools for the spraying because hollow
bones marked with pigment were discovered close by. Remember
that we must choose an answer that deals with the content of the sentences
that we are analyzing. When we look at the answer choices again, we should
see now that B is the answer. C, we eliminated; A, has nothing to do with the
information that we read; D has nothing to do with the information that we
read. There was no reference to fear in our sentences.

 
*If you chose A or D, then we are losing points for no reason. Remember to always
choose an answer that has content that comes from the sentences your analyzing
to find the answer.

Question 2

This is an inference question. We know this because it asks us what the reading
infers, implies, or suggests.

1)    First, we read and understand our question: what can be inferred
about the artists of most cave paintings other than those in
Lascaux. With this question, we notice that we must say something about
the artists of paintings that are not in Lascaux.

2)    Our key words: artists, cave paintings, other than those in
Lascaux.

3)    Our key words are in the second sentence: Unlike rock paintings
found in other parts of the world, these were not located near the
entrance to the cave or completely out in the open.

4)    Remember from the title that the whole reading is about cave paintings
in Lascaux and our sentence says that the paintings in Lascaux were not
located near the entrance unlike other paintings, so what's the opposite of
that? Paintings that are located near the entrance. We can therefore infer
that rock paintings found in other parts of the world were located near the
entrance. If that’s not quite clear read through this paragraph again to
ensure that it is.
5)    We check our answer choices and unfortunately no answer states our
inference clearly. We know we must continue reading and dig deeper. That
makes this a hard inference. If we are stressed for time, this might be a
question that we guess on and come back to later.

6)    The next sentence states: Instead, they are deep within the
caverns, quite a distance from the opening to the grotto and any
natural light. What can we infer from this sentence? We remember that
this describes paintings in Lascaux, so paintings that are not from Lascaux
are not like this: other rock paintings are not deep within the caverns and
they are not a distance from the opening and they are not a distance from
any natural light. If we check our answer choices, now we have enough
information to choose an answer. A doesn't work because we have not read
about this idea. B makes sense because we know that other cave paintings
were likely near the entrance and near natural light. C doesn't reference any
information that we read about and D doesn't reference any information that
we’ve read either; they are both wrong. B is our answer.

Question 3

This is an inference question. We know this because it asks us what the reading
infers, implies, or suggests.

1)    First, we read and understand our question: what is implied in the
theory about the fact that some paintings in Lascaux are drawn
directly over others. We are being asked what the significance is behind
the fact that some paintings are drawn over. We must understand this first
before we continue on to the next step.
2)    Our key words: some paintings, Lascaux, drawn directly over
others.

3)    The third sentence contains our key words and reads: And since some
paintings are drawn directly over others, scientists theorize that the
utility of each painting was over when the migration depicted had ended.

4)    So, what can we understand from the sentence about the paintings
drawn directly over others? All we understand is that the utility was over
when the migration it depicted ended. What does utility mean? It means
usefulness, so the usefulness of the painting was over when the migration
ended. Remember that our keywords are located in a clause that begins with
since. That means that our keywords are the reason for the scientist theory.
What does that mean? Since each painting was drawn over, scientists
theorize that the painted material was no longer useful. This
implies that if the paintings were useful, they wouldn’t have been drawn
over. With this information we look at our answer choices.

5)    A looks very good and is exactly what we expect. B doesn't look good
because we read nothing about location. (We read about location in the
next sentence, but we don't need to read the next sentence because the
sentence with our keywords points to an answer: A). C has keywords from
the sentence before, so for the same reason that B is wrong, C is wrong. D is
wrong because we didn’t read about variation from year to year in migration.

Question 4

This is an inference question. We know this because it asks us what the reading
infers, implies, or suggests.

 
1)    First, we read and understand our question: what can be inferred
from the presence of spears and arrows in some of the paintings.
This asks us what the presence of spear and arrows could mean.

2)    Our key words: presence, spears and arrows, some of the
paintings.

3)    We've read the sentence before and we read it again: The fact that in
several of the paintings the animals are portrayed as wounded by arrows
or spears gives particular credence to this theory.

4)    This means that the presence gives credits to this theory. What
can we infer? Without the arrows or spears, there would be less certainty
to this theory. Which theory? The one that we’ve been reading about in this
paragraph: paintings were meant to prepare hunters for the hunt. What else
does it mean? How did the animal get hit by an arrow or spear? The idea is
that it must have been thrown by a hunter. This implies that the animals in
the paintings were the animals that were hunted and attacked by the
hunters.

5)    If we check the answer choices, we find that the answer is B. A doesn't
make sense because there's no information about advanced weaponry. B
makes sense because the animals are attacked or preyed upon by the
hunters. C is wrong because the paragraph implies that the hunters had
great use for the paintings prior to the hunt as tools for preparation rather
than no use for them after the hunt. C is a common TOEFL wrong answer: it
is an opposite. D goes too far. This theory tells us that the paintings were
important, but to say that they were central rituals in the life of the tribe is
too strong. B is much closer to the reading and therefore a better answer.

Question 5
 

This is an inference question. We know this because it asks us what the reading
infers, implies, or suggests.

1)    First, we read and understand our question: what is implied about
the hunting done by the cave dwellers if the paintings were in fact
part of the ritual. This asks us to assume something: paintings were part
of the ritual. Then, from the paragraph, we must answer what that means
about the cave dwellers who hunted.

2)    Our key words: hunting, cave dwellers, paintings, part of the
ritual.

3)    We find our key words in the second sentence of the paragraph: The
theory states that the paintings were part of a ritual used to prepare the
hunters psychologically for the challenges they were about to face.

4)    From this we learn that if the paintings were part of a ritual, then we
know that the ritual was meant to prepare the hunters psychologically. What
does this tell us? It means that the hunters needed to be prepared mentally.
This implies that physical strength alone wasn't enough; the hunters also had
to be mentally prepared for their task (just like you have to be for the
TOEFL.) Does that give us an answer?

5)    A doesn't make sense because there's no concept of sustainability. B


doesn't make sense because there is no description of the weapons being
advanced. C may look like a good answer because it states it played a
vital role, and here it refers to the hunters and not the paintings, so it
creates a false meaning. This is a common play on words that the TOEFL will
use to create a wrong answer with. Look out for it. D is the best choice.
Physical strength was not the only factor that was important for successful
hunt; they needed to prepare mentally, which is a synonym of motivated
and focused. D restates our idea perfectly and is our answer.

Question 6

This is an inference question. We know this because it asks us what the reading
infers, implies, or suggests.

1)    First, we read and understand our question: what does the
information about the possible mythical meaning of the paintings
to cave dwellers imply about the cave dwellers. This asks us to infer
something about the cave dwellers if the theory of the mythical meaning is
true.

2)    Our key words: mythical meaning, paintings, cave dwellers.

3)    We go to paragraph 5 and search for keywords and find them in the first
sentence: A third opinion on the paintings offers the possibility that the
animals portrayed on the walls may have assumed some sort of
mythical meaning to the cave dwellers, either as ancient ancestors or as
protectors of the members of the tribe.

4)    We read the sentence and understand that if the theory is correct, these
cave dwellers thought the animals had a mythical meaning as either ancient
ancestors or protectors of the tribe. What does this mean? If something had a
mythical meaning, that implies that it was part of their culture, part of
something important to them. We infer that cave dwellers though that their
ancient ancestors or protectors were valuable to their society.
5)    With this information we read our answer choices. A make sense
because revered is another word for respect, which implies that the
ancestors were important to them. B is not supported and is just a random
wrong answer; it has our key word ancestors but there’s no information in
the reading about whether they were members of the tribe. C is wrong
because we read nothing about trance so far. D doesn't make sense because
they in the answer choice refers to the cave dwellers and the cave dwellers
didn't take on a mythical meaning; the animals did. A is our answer.

Question 7

This is an inference question. We know this because it asks us what the reading
infers, implies, or suggests.

1)    First, we read and understand our question: what can be inferred
about the scientific community's reaction to the paintings. This
asks us to understand something that comes from the reaction of the
scientists, so that is what we must analyze.

2)    Our key words: scientific community, reaction, paintings.

3)    We then read paragraph 6 and find our key words in the first sentence:
While these theories continue to compete for primacy in the scientific
community, and there is still heavy debate. Heavy debate and
compete for primacy represent controversy and argument.

4)    This tells us two things. First, it tells us that theories are competing to be
the major theory and second, it tells us that there is heavy debate within this
competition. What is the reaction of the community? Debate. What does this
imply? Scientists don’t agree, they argue their point. Why would they do
this? They believe their point is true and worth arguing, important enough to
argue about. You wouldn’t argue about something you don’t care about,
right? We have our inference. Armed with this information, we read the
answer choices looking for an answer that says the same information.

5)    A seems to illustrate this idea well; if something is still interesting, it


implies that it is still important or still has value. B is incorrect because
waned actually means decrease and the interest has not decreased; it’s still
important enough to argue about. C looks good but in comparison to A it is
not as good. We do not know that this is a major area of research but it is an
area of research that interests some; just because something is important to
you doesn’t mean that it’s important to everyone, right? D creates a false
meaning. We know the scientific community is engaged by the mystery, but
that doesn't mean that they are encouraged by it. Encouraged is not a
synonym of engaged. Therefore our correct answer is A.

   

Final Advice back to top

One of the toughest questions on the exam, inference tricks many students. Make sure
that with these questions, you do your best but don't let it or other questions that
challenge you steal your time and force you into a lower score. Keep your timing. That's
my final point for this. Review, learn, master but don't let too much time go by as you try
to answer this.

 Pronoun Test

Pronoun Strategy back to top


 

The other question type is pronoun. Pronoun questions don’t appear often and they
always ask you what a certain word refers to. Here’s an example.

The word “them” in paragraph 4 refers to:

Here’s the strategy.

1)    Determine whether the pronoun is singular or plural.

2)    Identify all of the nouns in the sentence before the pronoun that match in
number (singular or plural)

3)    Choose the one that fits the meaning of the sentence best.

4)    Find that noun among your answer choice and choose it.

Reading Practice back to top

Cave Paintings in Lascaux

The Lascaux Grotto, in Southwest France, is a series of narrow cave chambers that was
discovered by children playing in the 1940s. ❒ These caves contained an amazing find
for historians – prehistoric paintings of animals on the walls. ⠝’ These massive animals
are sometimes portrayed as following one after another in a long line, but on other
occasions, they are sideways on the wall and some even upside down. Paintings are
mostly done in reds, yellow, and brown, and feature such beasts as wild horses, bulls,
reindeer, bison, and mammoths. ❒ Scientists analyzed the substance of the “paint” and
discovered that the colors came from ocher and other iron oxides that had been ground
into a fine powder. ❒ This powder then appears to have been brushed or smeared onto
the cave walls in some cases, while in other cases the artists of these paintings seem to
have blown or sprayed the material onto the rock surface. In these cases, scientists
surmise that tubes made from hollow bone may have been used as tools for the spraying
because hollow bones marked with pigment were discovered close by.

The location of these particular paintings has become a source of mystery to the
scientific community. ❒ Unlike rock paintings found in other parts of the world, these
were not located near the entrance to the cave or completely out in the open. ❒ Instead,
they are deep within the caverns, quite a distance from the opening to the grotto and any
natural light. ❒ In addition, it seems likely that the drawers of these paintings did not
necessarily want them to be seen by others, as inhabitants of caves usually lived
relatively close to the entrances. ❒ The odd location of these particular paintings has
led to various theories as to why so many generations of artists chose to keep their work
so well hidden.

❒ Three opinions have been given by the scientific community as to the perplexing
origins and meaning of these paintings. ❒ The most straightforward of these
explanations is that the paintings are simply a record of the migrations of herds of local
animals. ❒ And since some paintings are drawn directly over others, scientists theorize
that the utility of each painting was over when the migration it depicted had ended. ⠝’
However, this theory fails to address the puzzling location of these paintings within the
caves, unless it was simply that the cave dwellers chose to commemorate the migrations
with ceremonies held in secret.

A second theory holds that these paintings were tied to hunting and they played a vital
role in a special preparation ceremony for the hunters. ❒ The theory states that the
paintings were part of a ritual used to prepare the hunters psychologically for the
challenges they were about to face. ❒ The fact that in several of the paintings the
animals are portrayed as wounded by arrows or spears gives particular credence to this
theory. ❒ And the overpainting is explained by the fact that the hunters had no use for
the paintings after the hunt itself, and thus felt free to cover them up in preparation for
their next foray into the field. ❒

A third opinion on the paintings offers the possibility that the animals portrayed on the
walls may have assumed some sort of mythical meaning to the cave dwellers, either as
ancient ancestors or as protectors of the members of the tribe. Supporters of this theory
point to two different pieces of evidence. First, there are the strange and cryptic
geometric shapes painted near the animals. Second, there is the remarkable absence of
men in any of the paintings. In the few cases that humans are depicted, they are very
crudely drawn with long and rigid bodies. ❒Some of the men are shown lying face
down while others have the heads of birds and other animals. ⠝’This suggests to
advocates of the theory that the people in the picture might be portrayed in a trance
state. ❒However, there is no definitive proof for this theory and it could just as easily
be that these bird-headed men are simply wearing masks for hunting.❒

While these theories continue to compete for primacy in the scientific community, and
there is still heavy debate. It is possible that there will never be a satisfactory answer to
this intriguing question. However, it is clear that the Lascaux paintings represent a great
artistic achievement for the time that will continue to engage researchers, whatever their
significance may be.

1)    The word “them” in paragraph 4 refers to:

a)    hunters

b)    arrows and spears

c)    paintings

d)    wounded animals

Power Review back to top

Just one example. There’s no reason to spend more time on it than this. Here’s the
strategy; you still have to memorize it.

1)    Determine whether the pronoun is singular or plural.

2)    Identify all of the nouns in the sentence before the pronoun that match in
number (singular or plural)

3)    Choose the one that fits the meaning of the sentence best.

4)    Find that noun among your answer choice and choose it.

 
Question 1

This is a pronoun question. We know this because we are asked what a word refers to.
We remember our strategy and follow it.

1)    First, we know that them is plural.

2)    Next, we look at the sentence before for plural nouns. In this sentence, we
actually have a complex sentence. This means that them could refer to plural
nouns within the same sentence because it is complex, which means we have two
or more sentences connected together. We only have two plural nouns before
them: hunters and paintings.

3)    As a result B and D are wrong. We should not have chosen them. The next
step is to identify the word that maintains an intelligent meaning in the sentence.
What would the hunters cover? Paintings makes more sense than hunters,
right?

4)    Our answer is C. This was a medium difficulty pronoun question.

Final Advice back to top

Don't stress over this question too much. It rarely appears. In your set of 3 (or 4
readings), you're likely to get only one example of this kind of question, maybe two, but
that's unlikely.

Plug-in Test
Plug-in Strategy back to top

The next question type is plug-in. Plug-ins ask you to place a sentence into one of four
points in a paragraph. Here’s an example.

Look at the four squares (❒) that indicate where the following sentence could be
added to the passage.

In fact, these water deposits have recently become the focus of


increased attention as a source of freshwater for the inhabitants
of desert areas.

Where would the sentence best fit?

These questions follow a few simple steps. Be aware though that if you have to get to the
last step, you’re dealing with a tough question that might be worth choosing quickly with
depending on our level.

1)    Read the sentence carefully to identify what it’s saying and the key words
within it.

2)    Read the paragraph and find the sentence that contains key words from the
plug-in.
For example, if you have the key word apples in your sentence. You read the
passage searching for apples. You find it between the answer choices A and B
and nowhere else. You know your sentence belongs in A or B.

3)    Use pronouns in your plug-in sentence to determine where your sentence
belongs.

4)    If that’s not enough. Identify words such as however, consequently, therefore,
etc. to determine whether your sentence continues the idea or begins the idea.

Continuing our apple example. If you see apples between A and B in the
paragraph, and your plug-in sentence begins with consequently. You’re
answer choice should be B because it is concluding the idea of apples and not
starting it.

5)    Finally, read through the paragraph with your answer in place and double
check that it makes sense.

Reading Practice back to top

Cave Paintings in Lascaux

The Lascaux Grotto, in Southwest France, is a series of narrow cave chambers that was
discovered by children playing in the 1940s. ❒ These caves contained an amazing find
for historians – prehistoric paintings of animals on the walls. ❒ These massive animals
are sometimes portrayed as following one after another in a long line, but on other
occasions, they are sideways on the wall and some even upside down. Paintings are
mostly done in reds, yellow, and brown, and feature such beasts as wild horses, bulls,
reindeer, bison, and mammoths. ❒ Scientists analyzed the substance of the “paint” and
discovered that the colors came from ocher and other iron oxides that had been ground
into a fine powder. ❒ This powder then appears to have been brushed or smeared onto
the cave walls in some cases, while in other cases the artists of these paintings seem to
have blown or sprayed the material onto the rock surface. In these cases, scientists
surmise that tubes made from hollow bone may have been used as tools for the spraying
because hollow bones marked with pigment were discovered close by.

The location of these particular paintings has become a source of mystery to the
scientific community. ❒ Unlike rock paintings found in other parts of the world, these
were not located near the entrance to the cave or completely out in the open. ❒ Instead,
they are deep within the caverns, quite a distance from the opening to the grotto and any
natural light. ❒ In addition, it seems likely that the drawers of these paintings did not
necessarily want them to be seen by others, as inhabitants of caves usually lived
relatively close to the entrances. ❒ The odd location of these particular paintings has led
to various theories as to why so many generations of artists chose to keep their work so
well hidden.

❒ Three opinions have been given by the scientific community as to the perplexing
origins and meaning of these paintings. ❒ The most straightforward of these
explanations is that the paintings are simply a record of the migrations of herds of local
animals. ❒ And since some paintings are drawn directly over others, scientists theorize
that the utility of each painting was over when the migration it depicted had ended. ❒
However, this theory fails to address the puzzling location of these paintings within the
caves, unless it was simply that the cave dwellers chose to commemorate the migrations
with ceremonies held in secret.

 
A second theory holds that these paintings were tied to hunting and they played a vital
role in a special preparation ceremony for the hunters. ❒ The theory states that the
paintings were part of a ritual used to prepare the hunters psychologically for the
challenges they were about to face. ❒ The fact that in several of the paintings the
animals are portrayed as wounded by arrows or spears gives particular credence to this
theory. ❒ And the overpainting is explained by the fact that the hunters had no use for
the paintings after the hunt itself, and thus felt free to cover them up in preparation for
their next foray into the field. ❒

A third opinion on the paintings offers the possibility that the animals portrayed on the
walls may have assumed some sort of mythical meaning to the cave dwellers, either as
ancient ancestors or as protectors of the members of the tribe. Supporters of this theory
point to two different pieces of evidence. First, there are the strange and cryptic
geometric shapes painted near the animals. Second, there is the remarkable absence of
men in any of the paintings. In the few cases that humans are depicted, they are very
crudely drawn with long and rigid bodies. ❒Some of the men are shown lying face down
while others have the heads of birds and other animals. ❒This suggests to advocates of
the theory that the people in the picture might be portrayed in a trance state.
❒However, there is no definitive proof for this theory and it could just as easily be that
these bird-headed men are simply wearing masks for hunting.❒

While these theories continue to compete for primacy in the scientific community, and
there is still heavy debate. It is possible that there will never be a satisfactory answer to
this intriguing question. However, it is clear that the Lascaux paintings represent a great
artistic achievement for the time that will continue to engage researchers, whatever their
significance may be.

 
1.    Look at the four squares (❒) that indicate where the following sentence could be
added to paragraph 1.

The creatures portrayed in these ancient paintings are


remarkable for their size, sometimes as much as 16 feet (almost
five meters) high.

Where would the sentence best fit?

2.    Look at the four squares (❒) that indicate where the following sentence could be
added to paragraph 2.

This must have presented some challenges for the artists, who
were forced to work in these tight areas without any sunlight to
aid their work.

Where would the sentence best fit?

3.    Look at the four squares (❒) in paragraph 3 that indicate where the following
sentence could be added to the passage.

 
Presumably, when the next migration season arrived, the artists
simply reused the existing space.

Where would the sentence best fit?

4.    Look at the four squares (❒) in paragraph 4 that indicate where the following
sentence could be added to the passage.

The hunters may have focused on them as a way of envisioning a


successful hunt.

Where would the sentence best fit?

5.    Look at the four squares (❒) in paragraph 5 that indicate where the following
sentence could be added to the passage.

The main argument for this is the fact that often those in trances
experience a sensation of weightlessness or believe that an
animal’s spirit is transferred to them during the trance.

Where would the sentence best fit?


 

Power Review back to top

Before we review, let’s make sure that we have the strategy mastered. If you don’t
master it now, don’t expect to do as well as you could on the next reading. Tap your full
potential; memorize the following steps 100%.

1)    Read the sentence carefully to identify what it’s saying and the key words
within it.

2)    Read the paragraph and find the sentence that contains key words from the
plug-in.

For example, if you have the key word apples in your sentence. You read the
passage searching for apples. You find it between the answer choices A and B
and nowhere else. You know your sentence belongs in A or B.

3)    Use pronouns in your plug-in sentence to determine where your sentence
belongs.

4)    If that’s not enough. Identify words such as however, consequently, therefore,
etc. to determine whether your sentence continues the idea or begins the idea.

Continuing our apple example. If you see apples between A and B in the
paragraph, and your plug-in sentence begins with consequently. You’re
answer choice should be B because it is concluding the idea of apples and not
starting it.

5)    Finally, read through the paragraph with your answer in place and double
check that it makes sense.
 

Question 1

This is a plug-in question. We know this because it asks us to place the sentence in
the best place in the paragraph. We remember our strategy and follow it.

1)    We read our sentence carefully: The creatures portrayed in these
ancient paintings are remarkable for their size, sometimes as
much as 16 feet high. We understand that this is talking about the large
size of the creatures in the paintings and we note our key words: creatures,
ancient paintings, size, 16 feet.

2)    As we read the sentences around our options in the paragraph, we see
synonyms of creatures between answer choice A, B, and C. We do not see
any reference to creatures before or after answer choice D. So, we should
eliminate that easily and quickly. If you chose D, you're not following our
strategy and losing points unnecessarily. Now, we have to find where we are
describing the size of the animals. The only word that is connected with size
is the word massive. Massive means very large and it appears between
answer choices B and C, so we can eliminate A. If we read carefully, we see
that there are two sentences between B and C. The word massive appears
only next to B and not C. Answer choice B is therefore our likely answer.

3)    We don’t need to use this because step 2 worked. This is an easy plug-in
question.

4)    We don’t need to use this because step 2 worked.

5)    When we read the sentences again to check to make sure that the
meaning is okay, it makes sense and we choose B comfortably.
 

Question 2

This is a plug-in question. We know this because it asks us to place the sentence in
the best place in the paragraph. We remember our strategy and follow it.

1)    We read our sentence carefully: This must have presented some
challenges for the artists, who were forced to work in these tight
areas without any sunlight to aid their work. We understand that this
sentence is closely connected to the sentence before it because of the
pronoun this. We also note our key words: challenges to the artists,
forced to work, tight areas, without sunlight, aid their work.

2)    As we read through the paragraph with an eye for our key words, we find
light between answer choices B and C. That makes answer choice A and D
incorrect. We should not have chosen them. Those are wrong answers that
we have to avoid because they are easy to avoid following our strategy.

3)    Our sentence gives us a big clue: this must have presented. Pronouns
usually go after the nouns that they refer to. Our answer should be C since
our sentence should continue the idea rather than start it because it has a
pronoun. B doesn’t look good anymore.

4)    We don’t need this step because step 3 is enough.

5)    We check by reading through the paragraph and ensure that our
sentence make sense in answer choice C, and it does.

 
Question 3

This is a plug-in question. We know this because it asks us to place the sentence in
the best place in the paragraph. We remember our strategy and follow it.

1)    We read our sentence carefully: Presumably, when the next
migration season arrived, the artists simply reused the existing
space. This sentence is very closely connected with the one before it with
respect to content. Our key words are: next migration season, artists,
reused the existing space.

2)    As we read through the paragraph, we see one of our key words,
migration, in the second, third, and fourth sentence. As a result, we
understand that A is not a good choice. B, C, and D present good choices
because they're next to those sentences with migration. To narrow down
the search, we look more carefully for our other key words and ideas: reused
existing space. In the third sentence we read: paintings are drawn
directly over others; that means that the artists (painters) reused the
existing space because they were painting in the same place again. This is an
excellent connection to our sentence but a bit hard to find. We can now
eliminate B. We are left with either C or D.

3)    One pronoun that you should remember but most students don’t
recognize is the. Remember that the is used to refer back to something that
was already mentioned. (I bought a car. The car was black.) Our sentence
states: the next migration. That doesn’t help us though because migration
appears before both C and D. We have another the reference though: the
existing space. The idea of space (drawn directly over) exists only between
C and D and since the functions as a pronoun that refers to something
already discussed. We know that our answer should be D.
4)    Presumably also helps because it means that the next sentence will
provide an assumption that follows from the sentence before. (He always
wins his games, so presumably he plays better than others.)

5)    If we replace our sentence and read the paragraph, the logic follows well.
D is our answer.

Question 4

This is a plug-in question. We know this because it asks us to place the sentence in
the best place in the paragraph. We remember our strategy and follow it.

1)    We read our sentence carefully: The hunters may have focused on
them as a way of envisioning a successful hunt. Our key words:
hunters, focused, envisioning, successful hunt.

2)    We read the paragraph and find hunters everywhere throughout the
paragraph. These are not good keywords. We must look more carefully.

3)    The, in the hunters, tells us that this is the second time that the word
hunters is mentioned and not the first for reasons discussed in the previous
question. That doesn’t help us much here though. Them refers to a plural
noun that comes from the sentence before and is what the hunters may have
focused on. What would hunters focus on? Paintings. Unfortunately,
paintings appear in every sentence. We move on.

4)    There’s no adverb to help us here.


5)    Now we find ourselves with a hard question. The key is: envisioning a
successful hunt. What does envision mean? It means to picture
something in your mind. Again, in your mind. You should see now that this
key word is connected with the word psychologically in the paragraph. If
you were able to draw this connection in step 2, that’s excellent. We could
have, but I placed this in step 5 because sometimes connections are difficult
to see and you must really struggle to find them. It may still be tough to
decide where our sentence belongs: B or C. What we do in such cases is
determine whether we can split a pair of sentences with our sentence. We
can’t choose B because before B, the sentence references theory and after B
the sentence references theory, so those two sentence are connected to each
other and we should not split them with ours because ours does not have the
word theory. If we do, we break the logic. It’s like writing apples, oranges,
apples, and oranges; instead of: apples, apples, oranges, and oranges. C is
our answer.

Question 5

This is a plug-in question. We know this because it asks us to place the sentence in
the best place in the paragraph. We remember our strategy and follow it.

1)    We read our sentence carefully: the main argument for this is the
fact that often those in trances experience a sensation of
weightlessness or believe that an animal’s spirit is transferred to
them during the trance. We see that this sentence provides an argument
for the sentence that comes before. Our key words are: trances, experiences,
sensation of weightlessness, animal’s spirit, transferred.
2)    We see trance in only one place in the paragraph, which is great for us.
The only possible answers are B and C. We should not have chosen A or D.
Those are wrong answers that we can avoid easily.

3)    Our sentence begins with the main argument for this is the fact
that. We have many pronouns that refer to the sentence before. As a result,
our sentence is continuing an idea and not introducing one. So, our answer
should be C.

4)    We don’t need to reference any connecting adverbs.

5)    We read the paragraph to check to make sure the meaning with our
sentence in C is okay and it is.

With several questions, such as this one, simply following our strategy will quickly
get us to the right answer. That’s why we focus so much on having you memorize
strategy to support you as you answer.

Final Advice back to top

  

This question is all about following strategy. Often, I see students get these questions
wrong because they make it more difficult than it should be. Just follow strategy
intelligently.

Paraphrase Test

Paraphrase Strategy back to top

 
You will identify paraphrase questions easily. These questions always ask which answer
choice states the information in a highlighted sentence the same way. Here’s an example
to clarify the idea.

Which of the sentences below best expresses the essential information in the
highlighted sentence in the passage? Incorrect choices change the meaning in
important ways or leave out essential information.

These questions challenge most students when they attack it without strategy. With our
strategy, these often become the easier, faster questions to answer, so pay careful
attention.

1)    Read the main independent clause and nothing else. Understand the subject,
verb, and object.

Break the sentence down piece by piece. They usually use complex sentences
with many parts--adjective clauses, adverb clauses, and so on. Don’t read the
whole highlighted sentence and then try to choose the answer. It’s the best
way to become confused.

2)    Read through the answer choices and eliminate any answer that does not state
this information exactly.

3)    Read the next clause in the highlighted sentence carefully.

4)    Read through the answer choices and eliminate any answer that does not state
this information exactly.

5)    Continue this process until only one answer is left.


 

This general explanation may seem a bit confusing, but once you try a few questions and
are guided through them by our system. It shouldn’t be a problem to follow this strategy
precisely.

Reading Practice back to top

Cave Paintings in Lascaux

The Lascaux Grotto, in Southwest France, is a series of narrow cave chambers that was
discovered by children playing in the 1940s. ❒ These caves contained an amazing find
for historians – prehistoric paintings of animals on the walls. ❒ These massive animals
are sometimes portrayed as following one after another in a long line, but on other
occasions, they are sideways on the wall and some even upside down. Paintings are
mostly done in reds, yellow, and brown, and feature such beasts as wild horses, bulls,
reindeer, bison, and mammoths. ❒ Scientists analyzed the substance of the “paint” and
discovered that the colors came from ocher and other iron oxides that had been ground
into a fine powder. ❒ This powder then appears to have been brushed or smeared onto
the cave walls in some cases, while in other cases the artists of these paintings seem to
have blown or sprayed the material onto the rock surface. In these cases, scientists
surmise that tubes made from hollow bone may have been used as tools for the spraying
because hollow bones marked with pigment were discovered close by.

The location of these particular paintings has become a source of mystery to the
scientific community. ❒ Unlike rock paintings found in other parts of the world, these
were not located near the entrance to the cave or completely out in the open. ❒ Instead,
they are deep within the caverns, quite a distance from the opening to the grotto and any
natural light. ❒ In addition, it seems likely that the drawers of these paintings did not
necessarily want them to be seen by others, as inhabitants of caves usually lived
relatively close to the entrances. ❒ The odd location of these particular paintings has led
to various theories as to why so many generations of artists chose to keep their work so
well hidden.

❒ Three opinions have been given by the scientific community as to the perplexing
origins and meaning of these paintings. ❒ The most straightforward of these
explanations is that the paintings are simply a record of the migrations of herds of local
animals. ❒ And since some paintings are drawn directly over others, scientists theorize
that the utility of each painting was over when the migration it depicted had ended. ❒
However, this theory fails to address the puzzling location of these paintings within the
caves, unless it was simply that the cave dwellers chose to commemorate the migrations
with ceremonies held in secret.

A second theory holds that these paintings were tied to hunting and they played a vital
role in a special preparation ceremony for the hunters. ❒ The theory states that the
paintings were part of a ritual used to prepare the hunters psychologically for the
challenges they were about to face. ❒ The fact that in several of the paintings the
animals are portrayed as wounded by arrows or spears gives particular credence to this
theory. ❒ And the overpainting is explained by the fact that the hunters had no use for
the paintings after the hunt itself, and thus felt free to cover them up in preparation for
their next foray into the field. ❒

A third opinion on the paintings offers the possibility that the animals portrayed on the
walls may have assumed some sort of mythical meaning to the cave dwellers, either as
ancient ancestors or as protectors of the members of the tribe. Supporters of this theory
point to two different pieces of evidence. First, there are the strange and cryptic
geometric shapes painted near the animals. Second, there is the remarkable absence of
men in any of the paintings. In the few cases that humans are depicted, they are very
crudely drawn with long and rigid bodies. ❒Some of the men are shown lying face down
while others have the heads of birds and other animals. ❒This suggests to advocates of
the theory that the people in the picture might be portrayed in a trance state.
❒However, there is no definitive proof for this theory and it could just as easily be that
these bird-headed men are simply wearing masks for hunting.❒

While these theories continue to compete for primacy in the scientific community, and
there is still heavy debate. It is possible that there will never be a satisfactory answer to
this intriguing question. However, it is clear that the Lascaux paintings represent a great
artistic achievement for the time that will continue to engage researchers, whatever their
significance may be.

In these cases, scientists surmise that tubes made from hollow bone may have been used
as tools for the spraying because hollow bones marked with pigment were discovered
close by.

1)    Which of the sentences below best expresses the essential information in the
highlighted sentence in the passage? Incorrect choices change the meaning in important
ways or leave out essential information.

a)    It is the conclusion of scientists that the tubes must have been made from
hollow bones because of the nearby pigment.
b)     Scientists believe that the hollow bones may have been used as tools to assist
in the spraying of the paint onto the walls due to their marked lack of pigment.

c)    According to scientists, the presence of pigment in the hollow bones found
near the paintings points to the possibility that they could have been used as tools
to spray the colors onto the walls.

d)    The discovery of the pigment in the hollow bones discounts the value of its use
as a rudimentary tool for spraying color onto the cave paintings.    

In addition, it seems likely that the drawers of these paintings did not necessarily want
them to be seen by others, as inhabitants of caves usually lived relatively close to the
entrances.

2)    Which of the sentences below best expresses the essential information in the
highlighted sentence in the passage? Incorrect choices change the meaning in important
ways or leave out essential information.

a)    Also, painters of cave drawings lived near the entrance of the cave, so they
drew paintings far to these openings to be hidden from view from the outside.

b)    It is probable that the creators of this art did not want others to easily discover
it because the drawings are far from where cave dwellers usually lived.

c)    The likelihood of the artists’ work being seen by others was enhanced by the
position of the painting close to where the inhabitants normally lived.

d)    The artists of the paintings didn’t want the art to be found, so they lived close
to the opening of the cave.

 
However, this theory fails to address the puzzling location of these paintings within the
caves, unless it was simply that the cave dwellers chose to commemorate the migrations
with ceremonies held in secret.

3)    Which of the sentences below best expresses the essential information in the
highlighted sentence in the passage? Incorrect choices change the meaning in important
ways or leave out essential information.

a)    Secret ceremonies may have been a vital part of the commemoration of the
migrations, but the cave dwellers didn’t address the location of the paintings
within the caverns.  

b)    However, the unusual location of the cave paintings is not adequately
explained by the theory, but it may have been simply that the residents of the caves
preferred that the commemorations be done in private.

c)    The theory fails to explain the odd location of the paintings, but secret
ceremonies give historians a hint of their purpose.

d)    The location of the painting is not addressed in this theory, but the cave
dwellers may have commemorated the locations with a ceremony hidden from
view.

The fact that in several of the paintings the animals are portrayed as wounded by arrows
or spears gives particular credence to this theory.

4)    Which of the sentences below best expresses the essential information in the
highlighted sentence in the passage? Incorrect choices change the meaning in important
ways or leave out essential information.
a)    Spears and arrows must have been the only weapons used in the hunt because
they are portrayed as wounding the animals in the pictures.

b)    This theory is supported by the presence of animals that have been injured by
human weapons in the cave paintings.

c)    Credence for this theory is portrayed because of the wounded animals in the
paintings.

d)    Without spears and arrows, according to this theory, the animals could never
have been wounded by the tribesmen.

However, there is no definitive proof for this theory and it could just as easily be that
these bird-headed men are simply wearing masks for hunting.

5)    Which of the sentences below best expresses the essential information in the
highlighted sentence in the passage? Incorrect choices change the meaning in important
ways or leave out essential information.

a)    This theory lacks true evidence, so the bird-headed men in the paintings could
be donning the masks for hunting.

b)    The presence of the hunting masks in the painting suggests to some that the
theory has no definitive proof.

c)    The theory has not been proven, and one equally plausible explanation is that
the birds’ heads in the painting were simply hunting masks worn by the men.

d)    Without definitive proof, this theory also holds that the bird-headed men in
the pictures often wore hunting masks.

 
Power Review back to top

We finished our answers and now we must check them and learn what we did well and
what we didn’t. Remember that the foundation of your success will come first from
mastering and memorizing the strategy below. Take time to memorize it if you haven’t
already.

1)    Read the main independent clause and nothing else. Understand the subject,
verb, and object.

Break the sentence down piece by piece. They usually use complex sentences
with many parts--adjective clauses, adverb clauses, and so on. Don’t read the
whole highlighted sentence and then try to choose the answer. It’s the best
way to become confused.

2)    Read through the answer choices and eliminate any answer that does not state
this information exactly.

3)    Read the next clause in the highlighted sentence carefully.

4)    Read through the answer choices and eliminate any answer that does not state
this information exactly.

5)    Continue this process until only one answer is left.

Question 1

 
This is a paraphrase question. We know this because it asks us to choose the
answer that restates the highlighted sentence best. We remember our strategy and
follow it.

1)    First, we identify the first main clause of the sentence focusing on the
subject, verb, and object: scientists surmise that. We know that this is
just saying that scientists believe something, so the object, which is a noun
clause, is very important for us to understand. Let's look at that: tubes
made from hollow bone may have been used as tools for the
spraying. Immediately, we should remember that the TOEFL often tricks
us by changing possibility to fact.

2)    We check our answers. A is wrong because it uses the words must have
been. If we chose A, then we are not remembering strategy to help us
improve our score. B looks good because it mentions our major points. C
looks good for the same reason. D doesn't mention scientists or the
possibility that tools may have been used. It's simply a confusing
answer that we should not choose. Now we have a 50% chance of getting the
answer right.

3)    Let's look at the next clause in our sentence: because hollow bones
marked with pigment were discovered close by. Immediately, we
understand that the TOEFL often plays with cause-and-effect relationships
(which means it will switch the word because with something else that
doesn’t work). Also, we know that the TOEFL often makes answers that are
opposite: B is a common wrong answer. B states: due to their marked
lack of pigment and our sentence says: marked with. C is the only
answer left. We check to make sure that it works just in case we made a
mistake, and it works and summarizes the sentence well.

 
Question 2

This is a paraphrase question. We know this because it asks us to choose the


answer that restates the highlighted sentence best. We remember our strategy and
follow it.

1)    First, we identify the first main clause of the sentence focusing on the
subject, verb, and object: it seems likely that. Immediately, we know that
the TOEFL often plays with possibility and fact to make incorrect answers.
Our highlighted sentence illustrates a possibility: it seems likely. We also
know that in this sentence the object, which is a noun clause, is very
important, so we read it: the drawers of these paintings did not
necessarily want them to be seen by others.

2)    Now, we check our answer choices. A states the information as a fact.
The content therefore doesn't matter much and we know that it is incorrect.
B mentions possibility and states that the artists did not want others to
easily discover the paintings, which restates the idea that the painters did not
necessarily want them to seen by others. C is tricky. It's just a reworded
sentence with our keywords, but it has the opposite meaning. It states that
the likelihood of the work being seen was enhanced, which is opposite (our
sentence says they might now have wanted the paintings to be seen by
others. C is wrong. D doesn't mention anything about possibility. D is
incorrect. As a result, we easily found B as the answer. Of course, we read the
whole sentence and our answer choice to make sure that we didn't make any
mistakes in our analysis, and we didn't.

This was an easy paraphrase that we should have chosen correctly.


 

Question 3

This is a paraphrase question. We know this because it asks us to choose the


answer that restates the highlighted sentence best. We remember our strategy and
follow it.

1)    First, we identify the first main clause of the sentence focusing on the
subject, verb, and object: this theory fails to address the puzzling
location of these paintings within the caves. We have a pronoun
within the main sentence, so we must understand what this pronoun means.
The theory mentioned in the sentence before is that the utility of each
painting was over when the migration depicted had ended. This
makes our main sentence fairly complicated and this question a little bit
challenging. As a result, let's be very exact as we examine our answer choices.

2)    Let's examine A. Our sentence states that the theory fails to address the
puzzling location. This sentence says that that cave dwellers didn't
address the location. It switches the subject (a common wrong answer on
the TOEFL) and therefore is wrong. Let's examine B. The unusual
location of the cave paintings is not adequately explained by the
theory states the same information as our main sentence, so we keep B. C
states that the theory fails to explain the odd location of the
paintings. This does have the same meaning as fails to address. We keep
C. Let's examine D. The location of the painting is not addressed in
this theory references our sentence’s information. When we look at these 3
answer choices carefully though, we should notice that B and C are better
than D. Did you see why? B states unusual location and C states odd
location. Odd and unusual are synonyms of puzzling. Those are good
answer choices. D just states the location, so it's missing one little word. D
doesn't look so good.

3)    Let's look at the second part of the sentence: unless it was simply
that the cave dwellers chose to commemorate the migrations with
ceremonies held in secret. What does unless mean? This is important
because we know that the TOEFL often changes the connection between
clauses within a sentence to confuse us. Think about this sentence: I will not
go unless you go. That means that if you go, I go. Unless introduces an
exception. This means that if the cave dwellers chose to commemorate the
migrations with ceremonies held in secret, then the theory does not fail to
address the puzzling location. Let's examine our answer choices. B states
that but it may have been simply that the residents of the caves
preferred that the commemorations be done in private. This
repeats the idea very well. B looks good. Let's look at C: but secret
ceremonies give historians a hint of their purpose. We have many
new keywords here historians, hints, purpose. C is not correct because
there are many words in the answer choice that are not in our sentence (this
is a common TOEFL wrong answer). Let's look at D. It states that the cave
dwellers may have commemorated the locations with a
ceremonies hidden from view. This is wrong because it changes
migrations to locations, and creates a false meaning. B is our answer.

Question 4

This is a paraphrase question. We know this because it asks us to choose the


answer that restates the highlighted sentence best. We remember our strategy and
follow it.

 
1)    First, we identify the first main clause of the sentence focusing on the
subject, verb, and object: the fact gives particular credence to this
theory. We must not get lost in the that clause that appeared in-between.
We know the subject is the fact and the verb is gives and it gives
credence, which means credit, strength, or support. What is the fact:
animals are portrayed as wounded by arrows or spears.

2)    We check the answer choices for the idea of the main sentence. A does
not look good because it uses our keywords but changes the meaning. A
states that spears and arrows must have been the only weapons.
That doesn't mean the same thing and we don’t know that from our
highlighted sentence: only makes it wrong. B states that this theory is
supported by something. Immediately we should be aware that this is
passive voice and we remember that one way the TOEFL makes answers
difficult to find is that it makes the correct answer or a wrong answer in
passive voice and switches the meaning in the process. The fact gives
credence to the theory turns into the theory is given credit by the
fact if we make it passive voice. B survives because it says the same thing. C
looks good for the same reason. D states that without spears and arrows
the animals could have never have been wounded by the
tribesmen. This uses our keywords but two points that we should know
make it wrong: first it creates more of an inference answer than a paraphrase
answer because we did not read this in our highlighted sentence. Remember
that paraphrase answers are restating the same information. The second
point is that it changes the meaning: the animals could never have
been wounded by the tribesmen. That is not what is stated in our
sentence at all. Don't get lost trying to argue for a wrong answer. Remember
that there should be one answer that is clearly correct. Both B and C say
essentially the same important meaning, but upon careful analysis we can
easily eliminate one. One restates more of the information, which makes it
correct. B states that the animals have been injured by human
weapons, which are arrows and spears, and C makes no reference to the
weapons. C is missing information; a common way to create a wrong
answer on the TOEFL. B contains more of the sentence and is therefore our
answer.

Question 5

This is a paraphrase question. We know this because it asks us to choose the


answer that restates the highlighted sentence best. We remember our strategy and
follow it.

1)    First, we identify the first main clause of the sentence focusing on the
subject, verb, and object: there is no definitive proof for this. We see
that there is a pronoun: this. We must read the sentence before to
understand this sentence’s meaning better. The sentence before states: the
theory that the people in the picture might be portrayed in a
trance state. And, that sentence is what this equals in our highlighted
sentence. Now, we have the complete meaning.

2)    We analyze each answer choice. A looks good: this theory lacks true
evidence. B looks good because it states the theory has no definitive
proof. C states the same thing the theory has not been proven. D is a
little strange; though it has our keywords, it changes the wording and creates
a strange answer; we don’t cross it off but won’t choose it unless the other
answers are 100% wrong.

3)    We look at the second clause to help us: and it could just as easily be
that these bird-headed men are simply wearing masks for
hunting. We recheck our answers and remember that these two pieces of
information are connected with and. That means that there are two different
facts in our highlighted sentence with no connection.

4)    A is a common TOEFL wrong answer because it changes the


relationship from and, equaling two facts, to so, which means as a result. A
is wrong. B does something similar by stating that the hunting masks
suggest no definitive proof. B is wrong. C contains our information and also
maintains the relationship with the two pieces of information with and. C
looks good. Since D is strange, we should be comfortable choosing C and
moving on. However, for the sake of clarity, let’s explain why D is 100%
wrong. D states that this theory also holds that the bird-headed men
in the pictures often wore hunting masks. The word also implies that
the theory essentially states two ideas and this is one of them. Our
highlighted sentence states that without definitive proof, our theory is no
longer strong and a different theory may be correct. As you can see it’s very
different to say one theory holds two ideas than to say that if one theory has
no proof, another theory may be true. D is wrong.

Final Advice back to top

With paraphrase questions, it's all about taking things step by step. Don't just read the
sentence and the answer choices back and forth. Follow strategy for this question as in
all questions. Once you master the technique, you'll find this a much easier question.

  Purpose Test

Purpose Strategy back to top

 
Purpose questions have two classes.

Type one

It asks you why the author mentions or states a word, phrase, or sentence in the
reading. These are the more common ones that follow strategy well and tend to be
easier. Here’s an example.

Why does the passage mention in paragraph 5 the fact that the exact extent
of groundwater resources is still unknown?

Here’s the idea behind the strategy. In standard, academic writing in English,
there stands a very strong structure. Titles provide the topic of the entire reading.
Paragraphs are groups of sentence that develop one piece of information or idea.
How do they develop these ideas? The first sentence represents the topic sentence
that outlines the focus of the paragraph. Every sentence after it continues the idea
by providing more detail. In other words, every sentence continues the idea before
it. That’s why are strategy is as follows.

1)    Read the question to understand what you’re being asked

2)    Identify the key words.

3)    Read the appropriate paragraph and find those key words or synonyms
of them.
4)    Read the sentence and understand that the purpose of the statement in
the question was to explain the content of that sentence.

For example, the question will ask, “Why does the author mention
apples?” You look for apples in the reading. Find it and then read the
sentence before and say to yourself, “the reason the author mentioned
apples was to explain the content in this sentence before.”

5)    Look for any adverbs that will help you to identify the relationship
between your key words and the content before it.

Continuing our example with apples. If between the two sentences, we


have the adverb “however.” The purpose of the author’s words was to
contrast the information in the sentence before.

6)    Now, choose an answer choice that reflects the information in the
sentence before.

Type two

It asks you to identify the purpose of a paragraph. These are difficult, but this
strategy will help you to attack them.

1)    Read the first sentence of the paragraph and choose an answer that
reflects that information.

2)    If that doesn’t give you an answer, read the last sentence of the
paragraph as well to further identify the focus of the paragraph and then
choose an answer that fits.
3)    If that doesn’t give you an answer, read the last sentence of the previous
paragraph. It may function as a transition sentence giving you a clue as to
what your paragraph is about.

4)    If that doesn’t work, understand the idea of the entire passage and find
how your paragraph fits with it. Then, choose an answer that reflects your
understanding.

This question can be either very easy or very tough depending on how many steps
you have to go down in our strategy. Do your best with it.

Reading Practice back to top

Cave Paintings in Lascaux

The Lascaux Grotto, in Southwest France, is a series of narrow cave chambers that was
discovered by children playing in the 1940s. ❒ These caves contained an amazing find
for historians – prehistoric paintings of animals on the walls. ❒ These massive animals
are sometimes portrayed as following one after another in a long line, but on other
occasions, they are sideways on the wall and some even upside down. Paintings are
mostly done in reds, yellow, and brown, and feature such beasts as wild horses, bulls,
reindeer, bison, and mammoths. ❒ Scientists analyzed the substance of the “paint” and
discovered that the colors came from ocher and other iron oxides that had been ground
into a fine powder. ❒ This powder then appears to have been brushed or smeared onto
the cave walls in some cases, while in other cases the artists of these paintings seem to
have blown or sprayed the material onto the rock surface. In these cases, scientists
surmise that tubes made from hollow bone may have been used as tools for the spraying
because hollow bones marked with pigment were discovered close by.

The location of these particular paintings has become a source of mystery to the
scientific community. ❒ Unlike rock paintings found in other parts of the world, these
were not located near the entrance to the cave or completely out in the open. ❒ Instead,
they are deep within the caverns, quite a distance from the opening to the grotto and any
natural light. ❒ In addition, it seems likely that the drawers of these paintings did not
necessarily want them to be seen by others, as inhabitants of caves usually lived
relatively close to the entrances. ❒ The odd location of these particular paintings has led
to various theories as to why so many generations of artists chose to keep their work so
well hidden.

❒ Three opinions have been given by the scientific community as to the perplexing
origins and meaning of these paintings. ❒ The most straightforward of these
explanations is that the paintings are simply a record of the migrations of herds of local
animals. ❒ And since some paintings are drawn directly over others, scientists theorize
that the utility of each painting was over when the migration it depicted had ended. ❒
However, this theory fails to address the puzzling location of these paintings within the
caves, unless it was simply that the cave dwellers chose to commemorate the migrations
with ceremonies held in secret.

A second theory holds that these paintings were tied to hunting and they played a vital
role in a special preparation ceremony for the hunters. ❒ The theory states that the
paintings were part of a ritual used to prepare the hunters psychologically for the
challenges they were about to face. ❒ The fact that in several of the paintings the
animals are portrayed as wounded by arrows or spears gives particular credence to this
theory. ❒ And the overpainting is explained by the fact that the hunters had no use for
the paintings after the hunt itself, and thus felt free to cover them up in preparation for
their next foray into the field. ❒

A third opinion on the paintings offers the possibility that the animals portrayed on the
walls may have assumed some sort of mythical meaning to the cave dwellers, either as
ancient ancestors or as protectors of the members of the tribe. Supporters of this theory
point to two different pieces of evidence. First, there are the strange and cryptic
geometric shapes painted near the animals. Second, there is the remarkable absence of
men in any of the paintings. In the few cases that humans are depicted, they are very
crudely drawn with long and rigid bodies. ❒Some of the men are shown lying face down
while others have the heads of birds and other animals. ❒This suggests to advocates of
the theory that the people in the picture might be portrayed in a trance state.
❒However, there is no definitive proof for this theory and it could just as easily be that
these bird-headed men are simply wearing masks for hunting.❒

While these theories continue to compete for primacy in the scientific community, and
there is still heavy debate. It is possible that there will never be a satisfactory answer to
this intriguing question. However, it is clear that the Lascaux paintings represent a great
artistic achievement for the time that will continue to engage researchers, whatever their
significance may be.

1.    In paragraph 1, why does the author mention the fact that pigment was found inside
the hollow bones?

a.    To provide evidence that the colors came from ocher

b.    To illustrate how the spraying process worked

c.    To explain why scientists reached their conclusion


d.    To emphasize the importance of pigment to the artistic process

2.    In paragraph 2, why does the author mention the locations of other rock paintings?

a.    To contrast these paintings with those found in Lascaux.

b.    To show how the Lascaux paintings represented an artistic advance.

c.    To illustrate the critical part natural light played in the paintings in Lascaux.

d.    To explain why scientists are unable to decipher the paintings

3.    In paragraph 5, why does the writer mention the unusual geometric shapes on the
walls?

a.    To illustrate a point the author is making about the quality of the drawings

b.    To show how these drawings were inferior to the those of the animals

c.    To give evidence that the drawings had a mythical significance to the cave
dwellers.

d.    To give an example of a type of drawing found outside the cave.

Power Review back to top

Don’t forget that it’s all about your comprehension, vocabulary, and strategy. Strategy is
the easiest to control, so memorize the steps and attack them diligently on your exam.
 

1)    Read the question to understand what you’re being asked

2)    Identify the key words.

3)    Read the appropriate paragraph and find those key words or synonyms of
them.

4)    Read the sentence and understand that the purpose of the statement in the
question was to explain the content of the information immediately preceding the
key words.

For example, the question will ask, “Why does the author mention apples?”
You look for apples in the reading. Find it and then read the sentence before
and say to yourself, “the reason the author mentioned apples was to explain
the content in this sentence before.”

5)    Look for any adverbs that will help you to identify the relationship between
your key words and the content before it. If you don’t find any, then you know the
purpose of the content was simply to explain the information that came before.

Continuing our example with apples. If between the two sentences, we have
the adverb “however.” The purpose of the author’s words was to contrast the
information in the sentence before.

6)    Now, choose an answer choice that reflects the information in the sentence
before.

Question 1

 
This is a purpose question. We know this because it asks us why the author stated,
mentioned, or said something. We remember our strategy and follow it.

1)    First, we identify the question and the keywords within it: why does
the author mention the fact that pigment was found inside the
hollow bones. We know that mentioning pigment inside hollow bone had a
purpose. Usually the purpose will fall into one of the following: explain,
support, or oppose.

2)    Our key words: pigment, inside, hollow bones.

3)    We see all of our keywords in the last sentence: In these cases, scientists
surmise that tubes made from hollow bone may have been used as tools for
the spraying because hollow bones marked with pigment were discovered
close by.

4)    Our sentence refers to the sentence before because of the pronoun:
these. We know that the sentence before refers to one method that paint
may have been placed on the wall. The sentence with our key words further
explains this by adding support for the theory.

5)    We read that the hollow bones were marked with pigment, and an
important word is before this sentence: because. Immediately, we should
see that the purpose of the author mentioning hollow bones marked
with pigment was to give a reason for something. A reason for what? We
read the information before our keywords: In these cases, scientists
surmise that tubes made from hollow bone may have been used as
tools for the spraying because hollow bones marked with pigment were
discovered close by. Our information gives reason or support to the scientists
surmising (which means inferring) an idea. We have our answer.
6)    We look at the answer choices. A is a bad choice because we read
nothing about ocher. We should not have chosen A. B doesn’t say what we
want it to. B is wrong. C looks good because we know that surmise means
to infer from our previous vocabulary question and it is also a synonym of
reached a conclusion. D is not a bad choice because it uses the keywords
in our sentence, but we know that because means reason and not
emphasis. C is our answer.

Question 2

This is a purpose question. We know this because it asks us why the author stated,
mentioned, or said something. We remember our strategy and follow it.

1)    First, we identify the question and the keywords within it: why does
the author mention the locations of other rock paintings. We know
that the purpose of this is likely to explain, oppose, or support something.

2)    Our key words: locations, other rock paintings.

3)    We learn about other rock paintings in the second sentence: unlike rock
paintings found in other parts of the world, these were not located near
the entrance to the cave or completely out in the open.

4)    In this sentence, we are comparing two things: rock paintings in Lascaux
(referenced by the word these) and rock paintings found in other parts of
the world.

5)    We have an adverb that gives us the context that this sentence: unlike.
Unlike means different from. This information about other rock
paintings is used to contrast or oppose information in the reading. We
know that its contrasting the paintings in Lascaux from our analysis in step
3.

6)    The only answer choice that works is A. Unlike does not mean show,
illustrate, or explain. As a result, answer choices B, C, and D are incorrect.

This was a clear and easy question. We need to have chosen A. If not, let’s review
strategy again.

Question 3

This is a purpose question. We know this because it asks us why the author stated,
mentioned, or said something. We remember our strategy and follow it.

1)    First, we identify the question and the keywords within it: why does
the writer mention the unusual geometric shapes on the walls. We
know that this information was likely used to explain, support, or oppose
something mentioned before.

2)    Our key words: unusual geometric shapes, walls.

3)    We find these key words in the third sentence of the paragraph: first,
there are the strange and cryptic geometric shapes painted near
the animals. We have our key words repeated exactly in a short sentence.

4)    We see the adverb first and know that this information is the first of
the series of facts that are likely meant to explain, oppose, or support
something. By reading the sentence before: supporters of this theory
point to two different pieces of evidence, we learn that this is the first
piece of evidence that supports a theory. We read the sentence before it to
learn what the theory is: a third opinion on the paintings offers the
possibility that the animals portrayed on the walls may have
assumed some sort of mythical meaning to the cave dwellers,
either as ancient ancestors or as protectors of the members of the
tribe. We find that the theory is that the animals portrayed on the walls
have a mythical meaning. So, the author stated geometric shapes as one
piece of evidence to support the theory that animals had a mythical
significance in the painting. We read our answer choices and look for it
there.

5)    A is wrong because we are not talking about the quality of the
drawings. B is wrong because we’re not talking about inferiority. C is
exactly what we want. D is wrong because it's not an example but a piece of
evidence that supports the theory. This is a relatively simple purpose
question. If we followed strategy, we should have chosen correctly.

Final Advice back to top

Short and simple: review and follow strategy. Even though vocabulary may challenge
you and the context may befuddle (confuse) you, this strategy will guide you to the right
choice.

 Summary Test

Summary Strategy back to top


 

Summaries are the last questions that you’ll attack and most students struggle with
them the most. We should discuss a few things about this question.

First, TOEFL preparation books often present two kinds of summary questions: one
asks you to choose 3 answer choices among 6 that complete a paragraph that
summarizes the reading. Here’s an example.

An introductory sentence for a brief summary of the passage is provided below.


Complete the summary by selecting the THREE answer choices that express the most
important ideas in the passage. Some sentences do not belong in the summary because
they express ideas that are not presented in the passage or are minor ideas in the
passage. This question is worth 2 points.

The unusual characteristics of the cave paintings at Lascaux have given rise to various
explanations as to the role they played in the lives of the cave dwellers.

This is the only kind of summary question that we have ever seen on the TOEFL. The
strategy to answer these questions quickly and accurately is intricate (which means
complex), so pay careful attention and review these steps often. They will only help
improve your score once you master them.

1)    First, review the structure of the reading on your own. Explain to yourself how
it is organized to remind you of the important information.
2)    Read the question carefully and see if the sentence in bold gives you a clue as
to what the answer choices might be.

If it states something about 3 theories, it’s easy to know that you must choose
3 answer choices among the 6 that describe 3 theories mentioned in the
reading.

3)    Read your first answer choice to understand what it states and find its key
words.

4)    Scan through the reading until you find the key words or synonyms of them
and read the sentences they are found in.

5)    Determine whether that answer choice is a summary, detail, or wrong answer.
Use the following method to do so.

(1)    If an answer choice is a summary, the answer choice restates


information found within the topic sentence of the paragraph, the first
sentence. That’s a good choice.

(2)    If the answer choice deals with a sentence that is not the first, it is a
detail. It is therefore not a good choice. You must choose answers that
summarize important points in the passage and not just minor points
mentioned in one sentence that is not the topic sentence.

(3)    If the answer choice describes content that is not found in the reading
or is wrong according to the reading, of course, eliminate it.

 
6)    After your analysis, you should be left with 3 summary answer choices. They
will be your answer.

7)     If you are left with 4 summary answer choices, read the sentence in bold in
the question and eliminate the answer choice that doesn’t directly deal with the
topic sentence.

8)     If you are left with 2 summary answer choices and 2 detail answer choices,
read the sentence in bold in the question and eliminate one of the detail answer
choices that deals with the topic sentence least.

However, books teach a second kind that divides the essay into two topics and asks you
to match different answer choices to the appropriate topic. Here’s an example.

Select the appropriate phrases from the answer choices and match them to the type of
organism to which they relate. TWO of the answer choices will NOT be used. This
question is worth 4 points.

For reasons that we won’t go into, this question is incredibly rare and easy strategy wise,
so we will not focus on it.

1)    Read an answer choice and identify the key words.

2)    Find the key words or synonyms of them in the reading.

3)    Read the sentence with your key word and determine what topic it belongs to.

4)    Match that answer choice to the appropriate topic.


 

Reading Practice

Cave Paintings in Lascaux

The Lascaux Grotto, in Southwest France, is a series of narrow cave chambers that was
discovered by children playing in the 1940s. ❒ These caves contained an amazing find
for historians – prehistoric paintings of animals on the walls. ❒ These massive animals
are sometimes portrayed as following one after another in a long line, but on other
occasions, they are sideways on the wall and some even upside down. Paintings are
mostly done in reds, yellow, and brown, and feature such beasts as wild horses, bulls,
reindeer, bison, and mammoths. ❒ Scientists analyzed the substance of the “paint” and
discovered that the colors came from ocher and other iron oxides that had been ground
into a fine powder. ❒ This powder then appears to have been brushed or smeared onto
the cave walls in some cases, while in other cases the artists of these paintings seem to
have blown or sprayed the material onto the rock surface. In these cases, scientists
surmise that tubes made from hollow bone may have been used as tools for the spraying
because hollow bones marked with pigment were discovered close by.

The location of these particular paintings has become a source of mystery to the
scientific community. ❒ Unlike rock paintings found in other parts of the world, these
were not located near the entrance to the cave or completely out in the open. ❒ Instead,
they are deep within the caverns, quite a distance from the opening to the grotto and any
natural light. ❒ In addition, it seems likely that the drawers of these paintings did not
necessarily want them to be seen by others, as inhabitants of caves usually lived
relatively close to the entrances. ❒ The odd location of these particular paintings has led
to various theories as to why so many generations of artists chose to keep their work so
well hidden.

❒ Three opinions have been given by the scientific community as to the perplexing
origins and meaning of these paintings. ❒ The most straightforward of these
explanations is that the paintings are simply a record of the migrations of herds of local
animals. ❒ And since some paintings are drawn directly over others, scientists theorize
that the utility of each painting was over when the migration it depicted had ended. ❒
However, this theory fails to address the puzzling location of these paintings within the
caves, unless it was simply that the cave dwellers chose to commemorate the migrations
with ceremonies held in secret.

A second theory holds that these paintings were tied to hunting and they played a vital
role in a special preparation ceremony for the hunters. ❒ The theory states that the
paintings were part of a ritual used to prepare the hunters psychologically for the
challenges they were about to face. ❒ The fact that in several of the paintings the
animals are portrayed as wounded by arrows or spears gives particular credence to this
theory. ❒ And the overpainting is explained by the fact that the hunters had no use for
the paintings after the hunt itself, and thus felt free to cover them up in preparation for
their next foray into the field. ❒

A third opinion on the paintings offers the possibility that the animals portrayed on the
walls may have assumed some sort of mythical meaning to the cave dwellers, either as
ancient ancestors or as protectors of the members of the tribe. Supporters of this theory
point to two different pieces of evidence. First, there are the strange and cryptic
geometric shapes painted near the animals. Second, there is the remarkable absence of
men in any of the paintings. In the few cases that humans are depicted, they are very
crudely drawn with long and rigid bodies. ❒Some of the men are shown lying face down
while others have the heads of birds and other animals. ❒This suggests to advocates of
the theory that the people in the picture might be portrayed in a trance state.
❒However, there is no definitive proof for this theory and it could just as easily be that
these bird-headed men are simply wearing masks for hunting.❒

While these theories continue to compete for primacy in the scientific community, and
there is still heavy debate. It is possible that there will never be a satisfactory answer to
this intriguing question. However, it is clear that the Lascaux paintings represent a great
artistic achievement for the time that will continue to engage researchers, whatever their
significance may be.

1)    Directions: An introductory sentence for a brief summary of the passage is provided
below. Complete the summary by selecting the THREE answer choices that express the
most important ideas in the passage. Some sentences do not belong in the summary
because they express ideas that are not presented in the passage or are minor ideas in
the passage.

The cave paintings at Lascaux represent a major artistic


achievement, but their unusual nature has led to different
theories about their purpose.

a)    One theory suggests that the men in the pictures were actually in trances.

b)    One theory holds that the animals in the paintings were mythical
representations of importance to the cave dwellers.
c)    According to one popular theory, the paintings might have been part of a ritual
that the tribe’s hunters used to prepare themselves for going out to hunt.

d)    Not every theory is able to explain why the paintings were done one on top of
the other in some cases.

e)    In one theory, the paintings may simply have been a way for the tribe to
document the migratory patterns of the herds of animals they encountered.

f)    The theory that has the most straightforward appeal involves the use of ocher
and other materials to create the colors.

2)    Directions: An introductory sentence for a brief summary of the passage is provided
below. Complete the summary by selecting the THREE answer choices that express the
most important ideas in the passage. Some sentences do not belong in the summary
because they express ideas that are not presented in the passage or are minor ideas in
the passage.

The Lascaux cave paintings are a major find in prehistoric art, but
their unique characteristics have led to a controversy regarding
their origins.

a)    It may have been that these paintings were merely a way for the tribe to keep
track of the movements of animals that passed by the cave.

b)    Perhaps the cave paintings had a mythical significance to the cave dwellers,
and they represented either ancestors or protectors of the tribe.
c)    A possible explanation for the paintings is that they were drawn as part of a
ritualistic preparation to motivate hunters to prepare for the challenges that faced
them.

d)    The paintings at Lascaux differ from others in the world, both in their content
and their location.

e)    The controversy remains unresolved, but this has not dampened interest in
the paintings, and in fact the mystery surrounding them makes them all the more
intriguing.

f)    A possible explanation for the men in the paintings is that they are wearing
hunting masks or ceremonial head dresses.

3)    Directions: An introductory sentence for a brief summary of the passage is provided
below. Complete the summary by selecting the THREE answer choices that express the
most important ideas in the passage. Some sentences do not belong in the summary
because they express ideas that are not presented in the passage or are minor ideas in
the passage.

The unusual characteristics of the cave paintings at Lascaux have


given rise to various explanations as to the role they played in the
lives of the cave dwellers.

a)    According to one theory, the overpainting occurred when the painting was no
longer needed by the tribe.

b)    It is a possibility that the tribe needed the information in the paintings to plan
the next season’s hunting.
c)    Some suggest that these paintings were in fact used as a way to psychologically
prepare hunters for their work.

d)    The paintings’ unusual location has caused some to speculate that the
paintings may have been part of some sort of secret ceremony.

e)    It could be that the paintings were used by the tribe to keep a record of the
migration of the animals in the paintings.

f)    There is a possibility that the animals depicted in the paintings had a mythical
significance to the tribe, either as ancestors or as protectors.

Power Review

At this point, you should see the pattern in choosing the right answer. The strategy is
intricate (which means complex), so it takes practice to master it, but once you do, you’ll
find these answers rather quickly. Take a moment to make sure that you’ve memorized
the strategy.

1)    First, review the structure of the reading on your own. Explain to yourself how
it is organized to remind you of the important information.

2)    Read the question carefully and see if the sentence in bold gives you a clue as
to what the answer choices might be.

If it states something about 3 theories, it’s easy to know that you must choose 3
answer choices among the 6 that describe 3 theories mentioned in the reading.

3)    Read your first answer choice to understand what it states and find its key
words.
4)    Scan through the reading until you find the key words or synonyms of them
and read the sentences they are found in.

5)    Determine whether that answer choice is a summary, detail, or wrong answer.
Use the following method to do so.

(1)    If an answer choice is a summary, the answer choice restates


information found within the topic sentence of the paragraph, the first
sentence. That’s a good choice. It’s also a good choice if it doesn’t discuss
points in the first sentence but in two or more found within the paragraph.

(2)    If the answer choice deals with one sentence that is not the first, it is a
detail. It is therefore not a good choice. You must choose answers that
summarize important points in the passage and not just minor points
mentioned in one sentence that is not the topic sentence.

(3)    If the answer choice describes content that is not found in the reading
or is wrong according to the reading, of course, eliminate it.

6)    After your analysis, you should be left with 3 summary answer choices. They
will be your answer.

7)     If you are left with 4 summary answer choices, read the sentence in bold in
the question and eliminate the answer choice that doesn’t directly deal with the
topic sentence.

8)     If you are left with 2 summary answer choices and 2 detail answer choices,
read the sentence in bold in the question and eliminate one of the detail answer
choices that deals with the sentence in bold in the question least.

Answer Key

 
Question 1

This is a summary question. We know this because it asks us to choose three


answer choices that complete a paragraph that summarizes the reading entirely.
We remember our strategy and follow it closely.

1)    First, we try to remember the major themes in the reading. This reading
stands as one that should be very easy for us to choose the correct answers
for. We know from the title that this is about the cave paintings in Lascaux
and we know from the first paragraph and the second paragraph that we are
focusing on why animals appear in the paintings. We know that the next
three paragraphs deal with three opinions for why animals were in the
paintings. The final paragraph summarizes the idea that we will never know
exactly why animals were painted as they were. This essay is very structured.
Now, we start to analyze each answer choice according to our strategy.

2)    We read the main sentence in the question for clues that usually aren't
helpful: the cave paintings at Lascaux represent a major artistic
achievement, but their unusual nature has led to different
theories about their purpose. This means that our answer choices will
likely reflect the different origins proposed within the article.

A: We should remember reading this and have an idea that is true. We


double check and find this information about trances in paragraph 5.
However, this appears in the middle of the paragraph, so we know that
this is a detail and therefore not a very good answer.
B: It has information that we should also remember reading. We find a
reference to mythical representations of importance to the cave
dwellers in paragraph 5. This information actually comes from the first
sentence of paragraph 5, so we know this answer represents the main
idea of the paragraph. We also know that A discussed information
from paragraph 5. We cannot choose two answers that are about the
same paragraph, and since B is better than A, we can eliminate A as an
answer choice.

C: We find reference to paintings being part of the ritual to prepare


hunters for the hunt in paragraph 4. We see this information in both
the first and second sentences of the paragraph. We therefore know
that this is a great answer because it deals with the topic sentence of
the paragraph.

D: We remember reading that one painting was done on top of


another in paragraph 3, but we do not read this information in any
other paragraph that deals with the theory for why the animals were in
the paintings. Therefore, answer choice D is correct. Only one theory
was able to explain partially why one painting was done over another
(paragraph 3), but the other two do not (paragraph 4 and 5). However,
this deals with only a minor point. Remember that the focus of these
theories is why animals were in the paintings. This is a little bit tricky,
but it is off topic and not related to the major idea of the reading. We
also understand this because this answer choice though it seems to deal
with many paragraphs does not deal with the content of the main
sentence.

E: We should remember reading about this theory about migratory


patterns from paragraph 3. This information comes from the second,
third, and fourth sentences of the paragraph. The first sentence is just a
general outline stating that we will learn about all three theories. As a
result, E is a good answer.
F: We read these keywords about a straightforward theory in
paragraph 3 and we read about ocher and other materials in paragraph
1. They have absolutely no relationship and this is a common wrong
answer for the TOEFL. We have keywords from different parts of the
passage that make the answer look correct, but they create a false
meaning that is not at all explained in the reading.

6)    Our answers are therefore B, C, and E. This was a medium to easy
difficulty summary question.

Question 2

This is a summary question. We know this because it asks us to choose three


answer choices that complete a paragraph that summarizes the reading entirely.
We remember our strategy and follow it closely.

1)    First, we try to remember the major themes in the reading, which we
already did in analyzing the previous summary question.

2)    We now review the main sentence in bold for clues: the Lascaux cave
paintings are a major find in prehistoric art, but their unique
characteristics have led to a controversy regarding their origins.
This means that our answer choices will likely reflect the different origins
proposed within the article.

 
A: Keep track of the movement of animals is a synonym for
migrations, which we should remember reading about in paragraph
3. This information appears in the second, third, and fourth lines, so
we know it represents an excellent summary of the paragraph and is
therefore a good answer.

B: We should remember reading about mythical significance in


paragraph 3. This information appears in the first sentence of the
paragraph, so we know it represents a good answer choice.

C: The information about the paintings being a ritualistic


preparation to motivate hunters to prepare for the
challenges that face them comes from paragraph 4. We know that
this information appears in the first sentence and in the second
sentence, so we know that C is a great answer.

D: This information about the Lascaux paintings being different from


others appears in the second sentence of the second paragraph only.
Therefore this answer is a detail answer choice.

E: This information about the controversy that still remains


about the paintings appears in paragraph 6. It represents the
information in the first and second sentences, so it is a strong answer
choice.

F: We remember reading about the possibility that the men in the


paintings were wearing hunting masks or head dresses in
paragraph 5. This information appears in the last sentence of the
paragraph and is therefore a detail. F represents detail answers in
summary questions that are usually wrong answers that we want to
avoid.

 
6)    We have 4 answer choices that reflect summary answer choices: A, B, C,
and E.

7)    In order to make the choice, we refer back to the main sentence of the
question: The Lascaux cave paintings are a major find in
prehistoric art, but their unique characteristics have led to a
controversy regarding their origins. We find that it asks us to find
sentences that deal with the paintings origins. So, we must avoid answers
that don't deal with the origins of the paintings. Which answer doesn't deal
with the possible origins or reasons for the paintings? E. Therefore, our
answers are A, B, and C.

Question 3

This is a summary question. We know this because it asks us to choose three


answer choices that complete a paragraph that summarizes the reading entirely.
We remember our strategy and follow it closely.

1)    First, we try to remember the major themes in the reading, which we
already did in analyzing the previous summary question.

2)    We now review the main sentence in bold for clues: the unusual
characteristics of the cave paintings at Lascaux have given rise to
various explanations as to the role they played in the lives of the
cave dwellers. We know that we’re choosing answers that deal with the
various explanations of the role of paintings to the cave dwellers. We now
analyze each answer according to our strategy.

 
A: Overpainting occurred when the painting was no longer
needed by the tribe is information that appears in paragraph 3. This
information appears in the middle of the paragraph and makes it a
detail answer choice that we will likely not choose.

b: We read about paintings used for hunting purposes in


paragraph 4. However, we do not read that the information in the
paintings were used to plan the next season’s hunting. We read
nothing about the paintings contributing to a plan. B is wrong. This is a
common way for the TOEFL to create a wrong answer. It provides key
words that you read and the general concept that may sound correct
but upon closer examination is false because there's a word in the
answer choice that is nowhere to be found in the reading. Don't be
tricked by this.

C: Again, we read about hunters using the paintings, but this time
we read that they were used to prepare hunters psychologically,
which we should remember is correct and in the first and second
sentence of paragraph 4. C represents a good answer.

D: We should remember reading about the unusual location


leading people to believe that the paintings were used for a
secret ceremony in paragraph 3 in the last sentence. It states that
the puzzling location might be explained by the fact that they were used
in secret ceremonies. This is not the main idea sentence (the first
sentence in case you forgot) and therefore D looks like a common
answer choice to avoid in the summary questions, a detail answer.

E: We should remember reading that a theory dealt with the idea that
the paintings were used to record the migration of animals in
paragraph 3. We read this in the second, third, and fourth sentences of
the paragraph. That makes E a good summary of the paragraph and
therefore a good answer.
F: This answer choice states that the paintings had a mythical
significance possibly related to ancestors or protectors of the
tribe, which represents the information in paragraph 5. This
information appears in the first sentence and so we know that F is a
good answer.

6)    The answer choices here are easy: C, E, and F.

Final Words

Congratulations! You did something extraordinary. You attacked or reading with such
detail that you were able to go in depth an answer 50 questions related to 700 words.
The Golan the purpose of this remember is to have you read the reading so much that
the content becomes so easy that you recognize that you have the ability to choose the
right answer as long as you develop your vocabulary and reading comprehension. Also,
since the information was so clear, the strategy should have become more comfortable
and you should have really been able to see the power. The only thing stopping you from
scoring high should now be complex sentences with vocabulary words that are difficult
to understand. Even then, you should still follow our strategy to be able to guess towards
the right choice when you have very difficult questions by following our keyword
analysis. Remember that we are always working to make and enhance our programs. We
we consider them excellent compared to others and we want to make them excellent by
far. We want to be the best TOEFL preparation program on the planet and we are
working hard to do so. This cannot be done without your contribution by letting us know
what was complex and difficult to understand what you're having trouble with, and what
we can do to help. So, don't hesitate to send us an e-mail about any of the questions that
you found difficult we reply almost instantaneously. Good luck with your studies and
keep on reading!
Improve Again

Let's Do It One More Time back to top

As you should know by now, my major focus as a teacher (good or bad though I find it
the second most important element that's made NoteFull successful and likely brought
you here) is clarity and mastery through repetition. Review these exercises. We can
always improve through them; these are the exercises that I use myself to ensure that
I'm always developing as well. They work for students of essentially all levels. Give them
your full energy and focus with a special emphasis on those exercises in the areas that
you have the most trouble with.

Expanding Your Vocabulary back to top

Your vocabulary is an important part of success on the reading section. Without a strong
vocabulary, it will be him to improve over time. The easiest way to know if your
vocabulary is an issue is to check the number of vocabulary words that you missed on
the previous reading.

The TOEFL asks you 4 vocabulary questions per reading on average. If you 1 or less
every reading, you are okay; your vocabulary is strong enough. However, if you miss 2 or
more questions, then we must focus on vocabulary or suffer a consistently lower score
than we deserve.

 
To improve your vocabulary, write down every word that you don't know in a reading
and master 7 of those words every day. To push your score ahead faster, focus on 14
words a day.

Another strong choice is to try our vocabulary system. Click here to learn more about it.
It's a fantastic system that we recommend strongly because every single word in that
system comes from actual TOEFL readings that have been released on the Internet-
based TOEFL. Also, we teach you these words in context. In addition, we teach the
correct pronunciation and focus your mastery on these words not only to develop your
reading ability through vocabulary, but also your speaking ability, listening ability, and
writing about.

Getting Faster Reading Speed back to top

One common difficulty among students is how quickly they can read through the
reading in order to attack the questions in 20 min. So, reading speed stands as an
incredibly important point to work on. Do you feel that the reading speed that you have
is too slow to answer all the questions?

You can improve your reading speed and maximize it to your level of fluency and even
go beyond it. But, it's going to take consistent work. Again, you have to do the following
exercise consistently every day in order to see benefits or else it's not going to be an
effective use of your time. Excited? Here it is.

First, you want to make sure that you have plenty of reading material to do this exercise.
You can find this by purchasing any National Geographic magazine or any Scientific
American magazine. You can also print several pages from National Geographic or
Scientific American online. Those are the only sources that I recommend. I don't
recommend TOEFL readings because you're going to go through them too quickly. I
don't recommend newspapers or magazines because they don't match the language of
the TOEFL very well.

Once you have, let's say, National Geographic, you want to begin the exercise. You will
need a stopwatch. Set your stopwatch for 1 min. When you start your stopwatch, your
goal is to read through any article in National Geographic (try to pick a very large one).
Once the timer buzzes and your 60 seconds are up, you want to stop and count the
number of lines that you were able to read. The reason why we count lines and not
words is that counting words is too time-consuming. Write this number down in your
notebook. For example, let's say that you did this exercise and read 12 lines in 60
seconds. That is your current ability and it's not strong enough, so we want to improve
it.

Reset your timer for 60 seconds. Now, your goal is to read twice as much information as
you did before in the same time. I know what you're thinking, “that's not possible.” It is.
But, you have to remember that you're pushing your limits and you're not going to read
at full comprehension. This exercise is meant to push you to read faster, so make sure to
follow it exactly. With a pen, mark the last line that you read in your article and then
count the number of lines that you need to read in this step and make another mark.
These marks will make it easier for you to know where you should read to. If we
continue our example, we have to read 24 lines. When you start the timer, read quickly.
Go through the words fast. Skip some that you think are not important. But, make sure
to continue to try to understand what you are reading. If you just go through the
information, it won't help.

 
Did you succeed? If you did, you can go onto the next step. However, if you did not
succeed, try again until you do. Never give up!

Now you should get the pattern. You want to, again, try to read twice as many lines as
you did. If you find that you don't have enough lines in your article, start a new article.
To clarify, if we continue our previous example, we now have to read 48 lines in 60
seconds. Make sure to try this again and again until you succeed. Remember that you
are not reading every single word. You are jumping around, skipping words, and reading
through lines quickly to grab as much content as you can. But, be aware that you only
have 60 seconds to complete your task.

It’s time to push ourselves. Let's do it one more time. If we continue our example, we
now must read 96 lines. If you really try, you're going to find that your brain is starting
to focus on nouns and verbs and nothing else. Also, it’ll focus on subjects and it's also
going to skip lines every now and then. You should still be able to grab some
information and understand the content that you’re reading. Well, you should at least a
little bit.

Now, here's the important part: we get to see if this exercise helped. If you followed it
carefully, you should see that it did; otherwise, do the whole thing again. Set your timer
for 60 seconds and read at full comprehension. You don't have any specific number of
lines that you want to read here but you want to read with full energy. Once the timer
buzzes, count the number of lines that you were able to read. Isn't it amazing? You
should now have read at least one line more than you did before and likely 2 to 6 lines
more. Remember that this is at full comprehension and you’ve just increased your
reading speed within 10 minutes. That’s serious progress.

 
This effect doesn't last forever. We lose our focus and our energy for reading very
quickly. That's why we have to do this exercise every single day before we start studying
and track our progress every time we do it in our notebooks to check that we’re always
improving and pushing ourselves.

Building Better Comprehension back to top

Through my years teaching, I've noticed that a lot of students don't really focus on
comprehension. They focus on everything else: vocabulary, reading speed, answer
choices, question types, strategy, and anything else you can name. However, I rarely see
a student ask me questions on how to improve his or her reading comprehension. And,
that's the most important element in your TOEFL success. High-level students who have
no strategy when reading but read a large amount of material develop the
comprehension that lets them score at a much higher level than many lower-level
students who know yeah I'm going to finish this really fast and you strategy very well.

Does this mean that the strategies that you're learning don't work? Of course not. The
point is to emphasize that deep down behind everything is comprehension. If you can’t
understand a sentence well, then you're not going to have an easy time answering a
question about that sentence regardless of how much strategy you know. Of course,
strategy will help you get the best result that you can though.

This pushes us to ask an important question: how can we improve our reading
comprehension? There are several ways. Some are very general; others are very specific.
Here's a system of reading that will push you to develop your reading comprehension.
 

First, you want to take any book that you find interesting. Don't focus on a book that you
believe will help you on the TOEFL. If you do that, you're not going to read with energy
and you're not going to benefit from it. Any book about any subject that's interesting to
you will work. A book is better than magazines or anything else because it forces you to
consistently read about the same information. This reinforces your vocabulary because
it will be recycled throughout the book. It also recycles the plot, which is the story of the
book, and that pushes you to consistently assess what it is that you're reading. In
simpler terms, a book is the most powerful way to improve your comprehension.

You should read consistently every day or every other day for about 30 to 60 min. Don’t
do it only for the TOEFL but for you in general and it will help your TOEFL score. Many
students who do this tell me how much it helps. Don't be one of the students who avoids
this exercise and in a few weeks communicates that he or she still hasn’t improved as
much as they needed to on the exam. You need to do this.

You can enhance your development of reading comprehension as you read by following
this next system of analysis. First, you always want to focus on the subject, verb, and
object of every sentence. The easiest way to do this is to have a pencil in your hand as
you read. Every time you have a subject, underline it; every time you have a verb,
underline it; and, every time you have an object, underline it. This will get pretty tedious
over time, so you don't have to do this all the time but you do want to do this for at least
10 to 20 min. of your reading time. This really helps because sometimes it will be hard
for you to find the subject, verb, or object and you will be forced to really think critically
about that sentence and its major meaning.

 
The next thing that you want to do is gradually work up your understanding of what it is
that you're reading. When you start reading you want to read with your pencil
underlining the subject, verb, and object as you just learned. You also want to have a
notebook next to you. In this notebook, restate every sentence that you read in one
paragraph. This doesn't mean that you copy it down but it means that you read this
sentence, digest it, and then rewrite it in your own words. Then, compare the sentence
in the reading to the sentence that you wrote down to make sure that the meaning is the
same. This is not easy. This will take time. But, you want to improve and this will
improve you.

You don't have to do this for every sentence that you read. That would be too much.
After the first paragraph or two, what you want to do instead of restating every sentence
is now restate every paragraph. You don't want to have more than one sentence
summarize one paragraph though. Write only one sentence to summarize a paragraph
that you read. By doing that, you'll force yourself to think about what the most
important piece of information is within that paragraph. That develops comprehension.

You want to do that for several paragraphs. To give you a good number, do this for about
10 paragraphs. Next, you want to summarize what you learned on an entire page if
you’re reading a book or if you're reading an article in a magazine or newspaper, you
want to summarize what you read in five paragraphs with one sentence. This is good if
you do it throughout the whole time you read but you don't have to; 20 minutes of your
reading time is fine. This focus on understanding the material that you're reading will
push her comprehension to the next level.

If you really want to be specific to the TOEFL, then do this exercise with TOEFL
readings that you have already read. Good luck!
 

Mastering Our Crucial Strategy back to top

This is the element that brought you to NoteFull. This is what you have been focusing on
for hours a day. Some of us have high levels of fluency and others of us have medium
levels of fluency but if we can make sure to follow strategy whenever we answer reading
questions, we will get the maximum points possible. And, if you really focus, you're
going to get a higher score than your current fluency level.

So, let me ask you a question: do you know all of the different question types on the
reading? Do you know all of the strategies associated with those question types? Do you
practice them consistently when you do a reading or do you still follow your own habit of
answering questions? Do you have them memorized so that you can bring them up in
your head quickly? Or, does it take you a lot of time to think of them and do you usually
forget one or two?

I cannot stress this enough: if you don't have these strategies and question types
mastered, you are not getting the highest score possible. It's that simple. If you want a
higher score, you must have these ideas consistent in your mind.

In my experience, the easiest way to develop mastery of something like this is to create
flash cards. One way to do that would be to take several index cards and on one side of
each index card write down a question type and on the other side, write down an
example question from an actual reading of that question type. Do that for every
question type. That is your first set of two flash cards.
 

For the next set of flashcards, you want to write, again, on one side, the question type.
Then, on the other side, you want to outline the strategy for it.

Review these flashcards as many times as it takes until it is easy in your mind to recall
not only the different question types but also the strategies associated with them. Why?
Because if you do not have them clear in your head right now when you're practicing in
the comfort of your own home, how easy do you think it's going to be under the stressful
conditions of the exam to remember them and get the highest score possible?

Improving on Your Most Difficult Questions back to top

One key quality that you need to make sure that you are improving in leaps instead of
very small steps is to know how to improve what is most difficult for you. This often
involves the hardest work because you have to focus on what is the most challenging and
as a result sometimes the most frustrating thing to improve.

For the reading section of the TOEFL, this means that you have to identify which
question type is holding you back the most. Do you find vocabulary questions the most
difficult? Do you find detail questions the most difficult? Whatever question is usually
the one that you miss, whatever question is the one that is the most difficult for you to
answer, whatever question takes the most time for you to answer, you want to identify it
and work on it.

 
How do you do that? After about two or three readings, you should know which question
type challenges you the most. Write this question type down in your notebook. Then, go
back to all of your old readings and circle all the questions that belong to this question
type. For example, let's say that my most difficult question is the detail question and I've
done about three readings in my time studying. I want to go through each reading and
circle every question that is a detail question. Then, what I want to do is write down or
copy and paste each question and the answer choices connected to it in one document. I
also want to copy and paste the paragraph that the answer is found in.

Let me clear this up a bit. Let's say that in one reading, question number 5 is a detail
question. I will copy question number 5 along with the answer choices and paste it in a
new word-processing document or write it down in my notebook if I have the time. In
the TOEFL, you know that the paragraph the answer is found in is always stated. So, if
question 5 says that the answer is in paragraph 4, then what I want to do is copy and
paste paragraph 4 above question number 5 in my document. Why are we doing this? If
you do this for every reading and for every question type that gives you difficulty, you're
going to have a powerful resource for practice.

Every time you study the reading section, review these questions. Review them following
strategy because if you don't then it's a useless exercise that's just going to push you to
memorize the right answer rather than develop a stronger answering ability.

The easiest way to do this is to go through the steps in choosing the right answer and
always review why each wrong answer is wrong. This will develop a much stronger skill
with the question type. It's a fantastic tool.

 
You can also do this not only for the question types that are most difficult but also for
any question in a reading that is really challenging for you. It's common that you'll find a
question in your practice that is very tough for you to understand even if you study the
answer key and even if you review it several times. That is one of the best tools for you to
improve with. Study that question every day until you really master it and understand
why the answer is what it is and why each wrong answer is incorrect. That will shoot up
not only your ability with strategy but your reading comprehension as well.

Remember that studying in this manner is incredibly important. Many students just go
through readings and practice and practice and don't improve because they're not really
developing new skills; they are just reviewing new vocabulary and building reading
comprehension little by little. Be as effective and improve as quickly as possible by
following these steps. Good luck!

Read more: https://www.notefull.com/myaccount.php?


w=10&productid=121&pgID=3925#ixzz1xmV9fOX6

Vous aimerez peut-être aussi